OB Exam 1

अब Quizwiz के साथ अपने होमवर्क और परीक्षाओं को एस करें!

A biophysical profile has been completed on a pregnant woman. The nurse interprets which score as normal? A. 9 B. 5 C. 3 D. 7

A. 9

The nurse is reviewing the monitoring strip of a woman in labor who is experiencing a contraction. The nurse notes the time the contraction takes from its onset to reach its highest intensity. The nurse interprets this time as which phase? A. increment B. decrement C. acme D. peak

A. increment

A nurse is assessing a rape survivor for post-traumatic stress disorder. The nurse asks the woman, "Do you feel as though you are reliving the trauma?" The nurse is assessing for which effect of the trauma? A. intrusive thoughts B. avoidance C. hyperarousal D. physical symptoms

A. intrusive thoughts

A client with genital herpes simplex infection asks the nurse, "Will I ever be cured of this infection?" Which response by the nurse would be most appropriate? A. "Once you have the infection, you develop an immunity to it." B. "There is no cure, but drug therapy helps to reduce symptoms and recurrences." C. "There is a new vaccine available that prevents the infection from returning." D. "All you need is a dose of penicillin and the infection will be gone."

B. "There is no cure, but drug therapy helps to reduce symptoms and recurrences."

Progesterone Only Oral Contraceptive contraindications:

Being overweight increases the chance of getting pregnant

A client states, "I think my waters broke! I felt this gush of fluid between my legs." The nurse tests the fluid with a Nitrazine swab and confirms membrane rupture if the swab turns: A. olive green. B. pink. C. blue. D. yellow.

C. blue.

A woman who is HIV-positive is receiving HAART and is having difficulty with compliance. To promote adherence, which area would be most important to assess initially? A. the woman's family and living arrangements B. the woman's financial situation and insurance C. the woman's beliefs and education D. the woman's activity level and nutrition

C. the woman's beliefs and education

After teaching a group of students about the different methods for contraception, the instructor determines that the teaching was successful when the students identify which of the following as a mechanical barrier method?

Condom Cervical cap Cervical sponge Diaphragm

Barrier contraceptives

Condom Diaphragm Cervical Cap Sponge

After discussing various methods of contraception with a client and her partner, the nurse determines that the teaching was successful when they identify which contraceptive method as providing protection against sexually transmitted infections (STIs)?

Condoms

The nurse notes persistent early decelerations on the fetal monitoring strip. Which action would the nurse do next? A. Perform a vaginal exam to assess cervical dilation and effacement. B. Stay with the client while reporting the finding to the primary care provider. C. Administer oxygen after turning the client on her left side. D. Continue to monitor the FHR because this pattern is benign.

D. Continue to monitor the FHR because this pattern is benign.

When assessing fetal heart rate, the nurse finds a heart rate of 175 bpm, accompanied by a decrease in variability and late decelerations. Which action would the nurse perform next? A. Have the woman change her position. B. Continue to monitor the pattern every 15 minutes. C. Administer oxygen. D. Notify the health care provider.

D. Notify the health care provider.

After teaching a community group about sexually transmitted infections, the nurse determines that the teaching was successful when the group identifies which statement as true? A. Individuals readily view the diagnosis of STI openly. B. Most individuals with STIs are over the age of 30. C. STIs have been addressed more on a global scale. D. STIs can affect anyone if exposed to the infectious organism.

D. STIs can affect anyone if exposed to the infectious organism.

A client is in the third stage of labor. Which finding would alert the nurse that the placenta is separating? A. umbilical cord shortens B. fetal head at vaginal opening C. mucous plug is expelled D. uterus becomes globular

D. uterus becomes globular

The nurse discusses various contraceptive methods with a client and her partner. Which method would the nurse explain as being available only with a prescription?

Diaphragm Pill Shot Patch Cervical Cap Implant IUD

Combined (progestin/estrogen) Oral Contraceptive side-effects

Dizziness, nausea, changes in mood, weight gain, changes in menstruation.

When discussing contraceptive options, which method would the nurse recommend as being the most reliable?

Intrauterine system IUD

combined oral contraceptive teaching and considerations:

Must be taken at the same time each day. Failure rate is increased with inconsistent dosing by the patient. Younger patients may not take as prescribed. Make suggestions for ensuring the patient takes meds (create a routine, set an alarm, etc.). Requires Rx. (parental consent under 18)

Hormonal contraceptives

Oral (Pill) Injectable Transdermal patch Vaginal Ring Implantable Intrauterine Emergency

A nurse is preparing a class for a group of women at a family planning clinic about contraceptives. When describing the health benefits of oral contraceptives, which of the following would the nurse most likely include?

Protection against pelvic inflammatory disease Reduced risk for endometrial/cervical/ovarian cancer Improvement in acne

A woman has opted to use the basal body temperature method for contraception. The nurse instructs the client that a rise in basal body temperature indicates which of the following?

Spinnbarkeit mucus - AKA Normal preovulation mucus

Progesterone Only Oral Contraceptives side-effects

Spotting or bleeding between periods, weight gain, breast tenderness.

A client is to receive an implantable contraceptive. The nurse describes this contraceptive as containing:

Synthetic Progestin

When developing a teaching plan for a couple considering contraception options, which of the following statements would the nurse include?

The best contraceptive is one that you will use correctly and consistently.

A client with a 28-day cycle reports that she ovulated on May 10. The nurse would expect the clients next menses to begin on a. May 24 b. May 26 c. May 30 d. June 1

a. May 24 For a woman with a 28-day cycle, ovulation typically occurs on day 14. Therefore, her next menses would begin 14 days later, on May 24

The nurse is preparing an outline for a class on the physiology of the male sexual response. Which event would the nurse identify as occurring first? a. Sperm emission b. Penile vasodilation c. Psychological release d. Ejaculation

b. Penile vasodilation

Estrogen increases the risk of

blood clots Thromboflabitis a risk

The nurse is creating a diagram that illustrates the components of the male reproductive system. Which structure would be inappropriate for the nurse to include as an accessory gland? a. Seminal vesicles b. Prostate gland c. Cowpers glands d. Vas deferens

d. Vas deferens The vas deferens is part of the ductal system of the male reproductive system responsible for transporting sperm from the epididymis. Accessory glands include the seminal vesicles, prostate gland, and Cowper's or bulbourethral glands.

After teaching a group of prospective new parents about the different perinatal education methods, the nurse determines that the teaching was successful when the parents identify which method as the Bradley method?

partner-coached method

Combined oral Contraceptive contraindications

women with: Breast feeding, high blood pressure, blood clots, heart attack, stroke, new vision problems, migraine with aura. Being overweight increases the chance of getting pregnant!

After the nurse teaches a client about ways to reduce the symptoms of premenstrual syndrome, which client statement indicates a need for additional teaching?

A) "I will make sure to take my estrogen supplements a week before my period." B) "I've signed up for an aerobic exercise class three times a week." C) "I'll cut down on the amount of coffee and colas I drink." D) "I quit smoking about a month ago, so that should help." Ans: A Feedback: Lifestyle changes such as exercising, avoiding caffeine, and smoking cessation are a key component for managing the signs and symptoms of premenstrual syndrome. Estrogen supplements are not used. If medication is necessary, NSAIDs may be used for painful physical symptoms; spironolactone may help with bloating and water retention.

A group of nurses is engaged in developing cultural competence. The students demonstrate achievement of this goal after developing which of the following?

A) Cultural knowledge B) Cultural skills C) Cultural encounter D) Cultural awareness Ans: C Feedback: The steps to developing cultural competence are cultural awareness, cultural knowledge, cultural skills, and, last, cultural encounter.

A nurse is assessing a family for barriers to health care. Which factor would the nurse identify to be most important?

A) Language B) Health care workers' attitudes C) Transportation D) Finances Ans: D Feedback: Financial barriers are one of the most important factors that limit care. Families may not have any medical insurance, may not have enough insurance to cover the services they need, or may not be able to pay for services. Language, health care workers' attitudes, and transportation are also barriers to health care but are not as fundamentally important as finances.

A woman using the cervical mucus ovulation method of fertility awareness reports that her cervical mucus looks like egg whites. The nurse interprets this as which of the following?

A) Spinnbarkeit mucus B) Purulent mucus C) Postovulatory mucus D) Normal preovulation mucus Ans: A Feedback: The client is describing spinnbarkeit mucus, the copious, clear, slippery, smooth, and stretchable mucus that occurs as ovulation approaches. Purulent mucus would be yellow or green and malodorous. Preovulation mucus is clear but not as copious, slippery, and stretchable.

While a nurse is obtaining a health history, the client tells the nurse that she practices aromatherapy. The nurse interprets this as which of the following?

A) Use of essential oils to stimulate the sense of smell to balance the mind and body B) Application of pressure to specific points to allow self-healing C) Use of deep massage of areas on the foot or hand to rebalance body parts D) Participation in chanting and praying to promote healing Ans: A Feedback: Aromatherapy involves the use of essential oils to stimulate the sense of smell to balance the mind, body, and spirit. Acupressure involves the application of pressure to specific points to restore balance and allow self-healing to take over. Reflexology is the use of deep massage on identified points of the foot or hand to scan and rebalance body parts that correspond to each point. Spiritual healing involves praying, chanting, presence, laying on of hands, rituals, and meditation to assist in healing.

When assessing a woman at follow-up prenatal visits, the nurse would anticipate which procedure to be performed? A. fundal height measurement B. hemoglobin and hematocrit C. fetal ultrasound D. urine for culture

A. fundal height measurement

Behavior contraceptives

Abstinence Fertility awareness methods Withdrawal (coitus interruptus) lactational amenorrhea

When palpating the fundus during a contraction, the nurse notes that it feels like a chin. The nurse interprets this finding as indicating which type of contraction? A. mild B. moderate C. intense D. strong

B. moderate

A nurse is working with a pregnancy woman to schedule follow-up visits for her pregnancy. Which statement by the woman indicates that she understands the scheduling? A. "Once I get to 28 weeks, I have to come twice a month." B. "I need to make visits every 2 months until I'm 36 weeks pregnant." C. "From now until I'm 28 weeks, I'll be coming once a month." D. "I'll make sure to get a day off every 2 weeks to make my visits."

C. "From now until I'm 28 weeks, I'll be coming once a month."

When planning the care of a woman in the active phase of labor, the nurse would anticipate assessing the fetal heart rate at which interval? A. every 45 to 60 minutes B. every 10 to 15 minutes C. every 15 to 30 minutes D. every 2 to 4 hours

C. every 15 to 30 minutes

After teaching a class at a local college campus on date rape, the nurse determines that the teaching was successful when the class identifies which substance as the most common date rape drug? A. liquid ecstasy B. ketamine C. gamma hydroxybutyrate D. rohypnol

D. rohypnol

A nurse is preparing a class for a group of young adult women about emergency contraceptives (ECs). Which of the following would the nurse need to stress to the group.

ECs are birth control pills in higher, more frequent doses ECs are not to be used in place of regular birth control ECs provide little (no) protection for future pregnancies.

Progesterone Only Oral Contraceptive teaching and considerations:

Good for breast feeding mothers Chance of getting pregnant greatly increased if taken just 3 hours late! Back-up method should be used for rest of month if missed.

Permantent Methods

Tubal ligation Vasectomy

A pregnant woman undergoes maternal serum alpha-fetoprotein (MSAFP) testing at 16 to 18 weeks gestation. Which of the following would the nurse suspect if the woman's level is decreased? a. Down syndrome b. Sickle-cell anemia c. Cardiac defects d. Open neural tube defect

a. Down syndrome Decreased levels might indicate Down syndrome or trisomy 18. Sickle cell anemia may be identified by chorionic villus sampling. MSAFP levels would be increased with cardiac defects, such as tetralogy of Fallot. A triple marker test would be used to determine an open neural tube defect.

A clients last menstrual period was April 11. Using Nageles rule, her expected date of birth (EDB. would be: a. January 4 b. January 18 c. January 25 d. February 24

b. January 18 April 11 + 7 days - 3 months

While talking with a woman in her third trimester, which behavior indicates to the nurse that the woman is learning to give of oneself? a. Showing concern for self and fetus as a unit. b. Unconditionally accepting the pregnancy without rejection c. Longing to hold infant d. Questioning ability to become a good mother

d. Questioning ability to become a good mother Learning to give of oneself would be demonstrated when the woman questions her ability to become a good mother to the infant. Showing concern for herself and fetus as a unit reflects the task of ensuring safe passage throughout pregnancy and birth. Unconditionally accepting the pregnnacy reflects the task of seeking acceptance of the infant by others. Longing to hold the infant relects the task of seeking acceptance of self in the maternal role to the infant.

A client states that she is to have a test to measure bone mass to help diagnose osteoporosis. The nurse would most likely plan to prepare the client for:

A) DEXA scan B) Ultrasound C) MRI D) Pelvic x-ray Ans: A Feedback: The client most likely will be having a DEXA scan, which is a screening test that calculates the mineral content of the bone at the spine and hip. Ultrasound, MRI, and a pelvic x-ray would be of little help in determining bone mass.

A pregnant woman asks the nurse, "I've heard that I should avoid eating certain types of fish. So what fish can I eat?" Which type of fish would the nurse recommend? Select all that apply. A. shark B. catfish C. salmon D. shrimp E. tilefish

B. catfish C. salmon D. shrimp

When a nurse suspects that a client may have been abused, the first action should be to: A. encourage the client to leave the batterer immediately. B. set up an appointment with a domestic violence counselor. C. ask the client about the injuries and if they are related to abuse. D. ask the suspected abuser about the victim's injuries.

C. ask the client about the injuries and if they are related to abuse.

A nurse is examining a female client and tests the clients vaginal pH. Which finding would the nurse interpret as normal? a. 4.5 b. 7 c. 8.5 d. 10

a. 4.5 The vagina has an acidic environment; therefore a pH of 4.5 would indicate an acidic environment. A pH of 7 is considered neutral; a pH above 7 is considered alkaline.

A nurse is describing the structure and function of the reproductive system to an adolescent health class. The nurse describes the secretion of the seminal vesicles as which of the following? a. Mucus-like b. Alkaline c. Acidic d. Semen

b. Alkaline The paired seminal vesicles secrete an alkaline fluid that contains fructose and prostaglandins. The fructose supplies energy to the sperm on its journey to meet the ovum, and the prostaglandins assist in sperm mobility. The Cowper's glands secrete a mucus-like fluid. The vagina is an acidic environment. Semen refers to the sperm-containing fluid.

A nurse is educating a client about a care plan. Which of the following would the nurse use to assess the client's learning ability?

A) "Did you graduate from high school; how many years of schooling did you have?" B) "Do you have someone in your family who would understand this information?" C) "Many people have trouble remembering information; is this a problem for you?" D) "Would you prefer that the doctor give you more detailed medical information?" Ans: C Feedback: It's appropriate to ask the client if he or she will have trouble remembering the information. Many clients have this problem. It removes any judgment or stereotypes regarding one's education level, understanding, or learning skills. Avoid giving information that uses a lot of medical language or jargon and use a simple, conversational style.

A pregnant client tells her nurse that she is interested in arranging a home birth. After educating the client on the advantages and disadvantages, which statement would indicate that the client understood the information?

A) "I like having the privacy, but it might be too expensive for me to set up in my home." B) "I want to have more control, but I am concerned if an emergency would arise." C) "It is the safest method for giving birth because there are no interferences." D) "The midwife is trained to resolve any emergency, and she can bring any pain meds." Ans: B Feedback: Home births have many advantages, such as having more control over the birth, being the least expensive option, creating a good relationship with a midwife, minimizing interferences, and having more flexibility in the comfort of your home. However, the limited availability of pain medication and danger to the mother and baby if an emergency arises are two of the main disadvantages. Thus, it is not considered the safest method for delivery

A nurse is teaching a group of college students about rape and sexual assault. The nurse determines that additional teaching is necessary based on which statements by the group? Select all that apply. A. Most victims of rape tell someone about it. B. Women have rape fantasies desiring to be raped. C. Medication and counseling can help a rape victim cope. D. Few women falsely cry "rape." E. A rape victim feels vulnerable and betrayed afterwards.

A. Most victims of rape tell someone about it. B. Women have rape fantasies desiring to be raped.

When developing a presentation for a local community organization on violence, the nurse is planning to include statistics on intimate partner abuse and its effects on children. When addressing these statistics, the nurse would identify what rate of the cases involving a parent being abused would the children be abused also? A. 1 in 10 B. 1 in 8 C. 1 in 3 D. 1 in 5

B. 1 in 8

A primiparous client is being seen in the clinic for her first prenatal visit. It is determined that she is 11 weeks pregnant. The nurse develops a teaching plan to educate the client about what she will most likely experience during this period. Which possible effect would the nurse include? A. ankle edema B. urinary frequency C. hemorrhoids D. backache

B. urinary frequency

A nurse is describing the various birth methods to pregnant couples. Which information would the nurse include as part of the Lamaze method? A. concentration on sensations while turning on to own bodies B. use of specific breathing and relaxation techniques C. interruption of the fear-tension-pain cycle D. focus on the pleasurable sensations of childbirth

B. use of specific breathing and relaxation techniques

A pregnant woman is scheduled for chorionic villus sampling. The nurse is describing the procedure and the potential for complications. When providing care to the client after the testing, the nurse would be alert for which complication as the most common? Select all that apply. A. spontaneous abortion B. vaginal bleeding C. rupture of membranes D. cramping E. hematoma

B. vaginal bleeding D. cramping

A pregnant woman is flying across the country to visit her family. After teaching the woman about traveling during pregnancy, which statement indicates that the teaching was successful? A. "I won't drink too much fluid so I don't have to urinate so often." B. "I'll sit in a window seat so I can focus on the sky to help relax me." C. "I'll get up and walk around the airplane about every 2 hours." D. "I'll do some upper arm stretches while sitting in my seat."

C. "I'll get up and walk around the airplane about every 2 hours."

A nurse is explaining the use of effleurage as a pain relief measure during labor. Which statement would the nurse most likely use when explaining this measure? A. "This technique focuses on manipulating body tissues." B. "The technique requires focusing on a specific stimulus." C. "The technique involves light stroking of the abdomen with breathing." D. "This technique redirects energy fields that lead to pain."

C. "The technique involves light stroking of the abdomen with breathing."

A nurse is teaching a pregnant client in her first trimester about discomforts that she may experience. The nurse determines that the teaching was successful when the woman identifies which discomfort as common during the first trimester? Select all that apply. A. leg cramps B. backache C. breast tenderness D. urinary frequency E. cravings

C. breast tenderness D. urinary frequency E. cravings

A nurse is providing care to a pregnant woman in labor. The woman is in the first stage of labor. When describing this stage to the client, which event would the nurse identify as the major change occurring during this stage? A. regular contractions B. fetal movement through the birth canal C. cervical dilation D. placental separation

C. cervical dilation

A 24-year-old client who is planning to become pregnant comes to the clinic for an evaluation. When assessing the client, which finding would alert the nurse to implement measures to reduce the client's risk for problems during pregnancy? Select all that apply. A. follows a vegetarian diet B. quit smoking 4 years ago C. has a BMI of 22 D. uses ibuprofen daily E. drinks wine 3 to 4 times/week

D. uses ibuprofen daily E. drinks wine 3 to 4 times/week

A pregnant woman comes to the clinic and tells the nurse that she has been having a whitish vaginal discharge. The nurse suspects vulvovaginal candidiasis based on which assessment finding? A. urinary frequency B. incontinence C. fever D. vaginal itching

D. vaginal itching

A biophysical profile has been completed on a pregnant woman. The nurse interprets which score as normal? a. 9 b. 7 c. 5 d. 3

a. 9 The biophysical profile is a scored test with five components, each worth 2 points if present. A total score of 10 is possible if the NST is used. Overall, a score of 8 to 10 is considered normal if the amniotic fluid volume is adequate. A score of 6 or below is suspicious, possibly indicating a compromised fetus; further investigation of fetal well-being is needed.

When assessing a woman in her first trimester, which emotional response would the nurse most likely expect to find? a. Ambivalence b. Introversion c. Acceptance d. Emotional lability

a. Ambivalence During the first trimester, the pregnant woman commonly experiences ambivalence, with conflicting feelings at the same time. Introversion heightens during the first and third trimesters when the woman's focus is on behaviors that will ensure a safe and healthy pregnancy outcome. Acceptance usually occurs during the second trimester. Emotional lability (mood swings) is characteristic throughout a woman pregnancy.

A group of students are reviewing the signs of pregnancy. The students demonstrate understanding of the information when they identify which as presumptive signs? (Select all that apply.) a. Amenorrhea b. Nausea c. Abdominal enlargement d. Braxton-Hicks contractions e. Fetal heart sounds

a. Amenorrhea b. Nausea

The nurse is assessing a 13-year-old girl who has had her first menses. Which of the following events would the nurse expect to have occurred first? a. Evidence of pubic hair b. Development of breast buds c. Onset of menses d. Growth spurt

b. Development of breast buds Pubertal events preceding the first menses have an orderly progression beginning with the development of breast buds, followed by the appearance of pubic hair, then axillary hair, then a growth spurt. Menses typically occurs about 2 years after the start of breast development.

Assessment of a pregnant woman reveals that she compulsively craves ice. The nurse documents this finding as which of the following? a. Quickening b. Pica c. Ballottement d. Linear nigra

b. Pica

A nurse is reviewing a journal article about vaccines used to prevent STIs. The nurse would expect to find information about vaccines for which STIs? Select all that apply. A. HPV B. HIV C. HAV D. HBV E. HSV

A. HPV C. HAV D. HBV

A couple comes to the clinic for a fertility evaluation. The male partner is to undergo a semen analysis. After teaching the partner about this test, which client statement indicates that the client has understood the instructions?

A) "I need to bring the specimen to the lab the day after collecting it." B) "I will place the specimen in a special plastic bag to transport it." C) "I have to abstain from sexual activity for about 1-2 days before the sample." D) "I will withdraw before I ejaculate during sex to collect the specimen." Ans: C Feedback: Semen analysis is the most important indicator of male fertility. The man should abstain from sexual activity for 24 to 48 hours before giving the sample. For a semen examination, the man is asked to produce a specimen by ejaculating into a specimen container and delivering it to the laboratory for analysis within 1 to 2 hours. When the specimen is brought to the laboratory, it is analyzed for volume, viscosity, number of sperm, sperm viability, motility, and sperm shape.

A group of nurses are researching information about risk factors for intimate partner violence in men. They are preparing a community presentation for the local health center. When describing the information, which factors would the nurses discuss as being associated with the relationship? Select all that apply. A. dysfunctional family system B. economic stress C. victim of childhood violence D. low academic achievement E. heavy alcohol consumption

A. dysfunctional family system B. economic stress E. heavy alcohol consumption

A nurse is preparing a presentation for a group of women at the clinic who have been diagnosed with genital herpes. Which factors would the nurse expect to include as possible precipitating factors for a recurrent outbreak? Select all that apply. A. emotional stress B. sexual intercourse C. use of corticosteroids D. exercise E. exposure to ultraviolet light

A. emotional stress B. sexual intercourse C. use of corticosteroids E. exposure to ultraviolet light

A nurse palpates a woman's fundus to determine contraction intensity. What would be most appropriate for the nurse to use for palpation? A. finger pads B. back of the hand C. finger tips D. palm of the hand

A. finger pads

A nurse is assessing a pregnant woman on a routine checkup. When assessing the woman's gastrointestinal tract, what would the nurse expect to find? Select all that apply. A. heartburn B. increased peristalsis C. nausea D. hyperemic gums E. complaints of bloating

A. heartburn C. nausea D. hyperemic gums E. complaints of bloating

A nurse is assessing a client diagnosed with pelvic inflammatory disease (PID). Which findings would the nurse most likely assess? Select all that apply. A. painful urination B. tenderness with cervical motion C. right upper quadrant pain D. negative pregnancy history E. oral temperature of 102 degrees F F. Clear vaginal discharge

A. painful urination B. tenderness with cervical motion D. negative pregnancy history E. oral temperature of 102 degrees F

A woman comes to the prenatal clinic suspecting that she is pregnant, and assessment reveals probable signs of pregnancy. Which finding would the nurse most likely assess? Select all that apply. A. positive pregnancy test B. ultrasound visualization of the fetus C. ballottement D. absence of menstruation E. auscultation of a fetal heart beat F. softening of the cervix

A. positive pregnancy test B. ultrasound visualization of the fetus C. ballottement D. absence of menstruation E. auscultation of a fetal heart beat

A nurse is working with a victim of intimate partner violence and helping her develop a safety plan. Which items would the nurse suggest that the woman take with her? Select all that apply. A. Social Security number B. driver's license C. cash D. health insurance cards E. phone cards

A. social security number B. driver's license C. cash D. health insurance cards

The nurse is assessing a pregnant woman in the second trimester. Which tasks would indicate to the nurse that the client is incorporating the maternal role into her personality? A. The client demonstrates unconditional acceptance without rejection. B. The woman acknowledges the fetus as a separate entity within her. C. The woman demonstrates concern for herself and her fetus as a unit. D. The client identifies what she must give up to assume her new role.

B. The woman acknowledges the fetus as a separate entity within her.

After teaching a refresher class to a group of prenatal clinic nurses about pregnancy, insulin, and glucose, the nurse determines that additional teaching is needed when the group identifies which hormone as being involved with opposing insulin? A. estrogen B. aldosterone C. prolactin D. progesterone

B. aldosterone

Assessment of a woman in labor reveals cervical dilation of 3 cm, cervical effacement of 30%, and contractions occurring every 7 to 8 minutes, lasting about 40 seconds. The nurse determines that this client is in: A. transition phase of the first stage. B. latent phase of the first stage. C. perineal phase of the second stage. D. active phase of the first stage.

B. latent phase of the first stage.

A woman suspecting she is pregnant asks the nurse about which signs would confirm her pregnancy. The nurse would explain that which sign would confirm the pregnancy? A. morning sickness B. palpable fetal movement C. abdominal enlargement D. absence of menstrual period

B. palpable fetal movement

A nurse is working with a victim of violence to develop a safety plan. The nurse teaches the client about the necessary items to take with her when leaving. The nurse determines that additional teaching is needed when the client identifies which items? Select all that apply. A. health insurance cards B. phone cards C. cash D. most of her clothing E. photo ID

B. phone cards D. most of her clothing

A nurse is working with a woman who is a victim of violence. Which intervention would be most important for this client? A. documenting the abuse B. providing for the client's safety C. reassuring her she is not alone D. educating about the cycle of violence

B. providing for the client's safety

While talking with a woman in her third trimester, which behavior indicates to the nurse that the woman is learning to give of oneself? A. unconditionally accepting the pregnancy without rejection B. questioning ability to become a good mother C. showing concern for self and fetus as a unit D. longing to hold infant

B. questioning ability to become a good mother

Which positions would be most appropriate for the nurse to suggest as a comfort measure to a woman who is in the first stage of labor? Select all that apply. A. closed knee-chest position B. straddling with forward leaning over a chair C. rocking back and forth with foot on chair D. walking with partner support E. supine with legs raised at a 90-degree angle

B. straddling with forward leaning over a chair C. rocking back and forth with foot on chair D. walking with partner support

A mother brings her 12-year-old daughter in for well-visit checkup. During the visit, the nurse is discussing the use of prophylactic HPV vaccine for the daughter. The mother agrees, and the daughter receives her first dose. The nurse schedules the daughter for the next dose, which would be given at which time? A. in 1 month B. in 3 months C. in 4 months D. in 2 months

D. in 2 months

A client comes to the prenatal clinic for her first visit. When determining the client's estimated due date, the nurse understands what which method is the most accurate? A. Nagele's rule B. gestational wheel C. birth calculator D. ultrasound

D. ultrasound

When describing the ovarian cycle to a group of students, which phase would the instructor include? a. Luteal phase b. Proliferative phase c. Menstrual phase d. Secretory phase

a. Luteal phase The ovarian cycle consists of three phases: the follicular phase, ovulation, and the luteal phase. The endometrial cycle includes the proliferative phase, menstrual phase, and secretory phase.

A nurse is discussing fetal development with a pregnant woman. The woman is 12 weeks pregnant and asks, "Whats happening with my baby?" Which of the following would the nurse integrate into the response? (Select all that apply.) a. Continued sexual differentiation b. Eyebrows forming c. Startle reflex present d. Digestive system becoming active e. Lanugo present on the head

a. Continued sexual differentiation d. Digestive system becoming active At 12 weeks, sexual differentiation continues and the digestive system shows activity. Eyebrows form and startle reflex is present between weeks 21 and 24. Lanugo on the head appears about weeks 13-16

A group of students are reviewing information about genetic inheritance. The students demonstrate understanding of the information when they identify which of the following as an example of an autosomal recessive disorder? (Select all that apply.) a. Cystic fibrosis b. Phenylketonuria c. Tay-Sachs disease d. Polycystic kidney disease e. Achondroplasia

a. Cystic fibrosis b. Phenylketonuria c. Tay-Sachs disease Examples of autosomal recessive disorders include cystic fibrosis, phenylketonuria, and Tay-Sachs disease. Polycystic kidney disease and achondroplasia are examples of autosomal dominant diseases.

When describing genetic disorders to a group of childbearing couples, the nurse would identify which as an example of an autosomal dominant inheritance disorder? a. Huntington's disease b. sickle cell disease c. Phenylketonuria d. Cystic fibrosis

a. Huntington's disease Huntington's disease is an example of an autosomal dominant inheritance disorder. Sickle cell disease, phenylketonuria, and cystic fibrosis are examples of autosomal recessive inheritance disorders.

A nurse is conducting a class for a group of teenage girls about female reproductive anatomy and physiology. Which of the following would the nurse include as an external female reproductive organ? Select all that apply. a. Mons pubis b. Labia c. Vagina d. Clitoris e. Uterus

a. Mons pubis b. Labia d. Clitoris

A woman just delivered a healthy term newborn. Upon assessing the umbilical cord, the nurse would identify which of the following as normal? (Select all that apply.) a. One vein b. Two veins c. One artery d. Two arteries e. One ligament

a. One vein d. Two arteries The normal umbilical cord contains one large vein and two small arteries.

After teaching a group of students about fetal development, the instructor determines that the teaching was successful when the students identify which of the following as essential for fetal lung development? a. Umblilical cord b. Amniotic fluid c. Placenta d. Trophoblasts

b. Amniotic fluid Amniotic fluid is essential for fetal growth and development, especially fetal lung development. The umbilical cord is the lifeline from the mother to the growing embryo. The placenta serves as the interface between the mother and developing fetus. It secretes hormones and supplies the fetus with nutrients and oxygen needed for growth. The trophoblasts differentiate into all the cells that form that placenta.

A nursing instructor is describing the hormones involved in the menstrual cycle to a group of nursing students. The instructor determines the teaching was successful when the students identify follicle-stimulating hormone as being secreted by which of the following? a. Hypothalamus b. Anterior pituitary gland c. Ovaries d. Corpus luteum

b. Anterior pituitary gland Follicle-stimulating hormone and luteinizing hormone are secreted by the anterior pituitary gland. The hypothalamus secretes gonadotropin-releasing hormone. The ovaries secrete estrogen. The corpus luteum secretes progesterone.

A nurse is developing a teaching plan about nutrition for a group of pregnant women. Which of the following would the nurse include in the discussion? (Select all that apply.) a. Keep weight gain to 15 lb b. Eat three meals with snacking c. Limit the use of salt in cooking d. Avoid using diuretics e. Participate in physical activity

b. Eat three meals with snacking d. Avoid using diuretics e. Participate in physical activity

During a prenatal class for a group of new mothers, the nurse is describing the hormones produced by the placenta. Which of the following would the nurse include? (Select all that apply.) a. Prolactin b. Estriol c. Relaxin d. Progestin e. Human chorionic somatomammotropin

b. Estriol c. Relaxin d. Progestin e. Human chorionic somatomammotropin Estriol, relaxin, progestin, and human chorionic somatomammotropin are secreted by the placenta. Prolactin is secreted after delivery for breastfeeding.

A woman comes to the clinic for an evaluation. During the visit, the woman tells the nurse that her menstrual cycles have become irregular. "I've also been waking up at night feeling really hot and sweating." The nurse interprets these findings as which of the following? a. Menopause b. Perimenopause c. Climacteric d. Menarche

b. Perimenopause

A nurse is assessing a child with Klinefelter's syndrome. Which of the following would the nurse expect to assess? (Select all that apply.) a. A needle will be inserted directly into the fetus's umbilical vessel. b. You'll have an intravaginal ultrasound to measure fluid in the fetus. c. The doctor will take a sample of fluid from your bag of waters. d. A small piece of tissue from the fetal part of the placenta is taken.

b. You'll have an intravaginal ultrasound to measure fluid in the fetus. Fetal nuchal translucency testing involves an intravaginal ultrasound that measures fluid collection in the subcutaneous space between the skin and cervical spine of the fetus. Insertion of needle into the fetus's umbilical vessel describes percutaneous umbilical blood sampling. Taking a sample of fluid from the amniotic sac (bag of waters) describes an amniocentesis. Obtaining a small tissue specimen from the fetal part of the placenta describes chorionic villus sampling.

Which of the following would the nurse include when teaching a pregnant woman about chorionic villus sampling? a. The results should be available in about a week. b. You'll have an ultrasound first and then the test. c. Afterwards, you can resume your exercise program. d. This test is very helpful for identifying spinal defects.

b. You'll have an ultrasound first and then the test. With CVS, an ultrasound is done first to localize the embryo. Results are usually available within 48 hrs. After the procedure, the woman should refrain from any strenuous activity for the next 48 hrs. CVS can be used to detect numerous genetic disorders but not neural tube (spinal) defects

The nurse is developing a presentation for a community group of young adults discussing fetal development and pregnancy. The nurse would identify that the sex of offspring is determined at the time of: a. meiosis b. fertilization c. Formation of morula d. Oogenesis

b. fertilization Sex determination occurs at the time of fertilization. Meiosis refers to cell division resulting in the formation of an ovum or sperm with half the number of chromosomes. The morula develops after a series of our cleavages following the formation of the zygote. Oogenesis refers to the development of a mature ovum, which has half the number of chromosomes.

A nurse measures a pregnant woman's fundal height and finds it to be 28 cm. The nurse interprets this to indicate which of the following? a. 14 weeks gestation b. 20 weeks gestation c. 28 weeks gestation d. 36 weeks gestation

c. 28 weeks gestation Typically, the height of the fundus is measured when the uterus arises out of the pelvis to evaluate fetal growth. At 12 week's gestration the fundus can be palated at the symphysis pubis. At 16 weeks' gestration the fundus is midway between the symhysis and the umbilicus. At 20 weeks the fundus can be palpated at the umbilicus and measures approximately 20 cm from the syphysis pubis.

A client who is 4 months pregnant is at the prenatal clinic for her initial visit. Her history reveals she has 7-year-old twins who were born at 34 weeks gestation, a 2-year old son born at 39 weeks gestation, and a spontaneous abortion 1 year ago at 6 weeks gestation. Using the GTPAL method, the nurse would document her obstetric history as: a. 3 2 1 0 3 b. 3 1 2 2 3 c. 4 1 1 1 3 d. 4 2 1 3 1

c. 4 1 1 1 3 Gravida: number of pregnancies: 4 T: number of term 1 P number of preterm 1 A: number of ended prior to age of 20 weeks 1 L: number of living children 3

A woman is at 20 weeks gestation. The nurse would expect to find the fundus at which of the following? a. Just above the symphysis pubis b. Mid-way between the pubis and umbilicus c. At the level of the umbilicus d. Mid-way between the umbilicus and xiphoid process

c. At the level of the umbilicus

A gravida 2 para 1 client in the 10th week of her pregnancy says to the nurse, "I've never urinated as often as I have for the past three weeks". Which response would be most appropriate for the nurse to make? a. Having to urinate so often is annoying. I suggest that you watch how much fluid you are drinking and limit it. b. You shouldn't be urinating this frequently now; it usually stops by the time you're eight weeks pregnant. Is there anything else bothering you? c. By the time you are 12 weeks pregnant, this frequent urination should no longer be a problem, but it is likely to return toward the end of your pregnancy. d. Women having their second child generally don't have frequent urination. Are you experiencing any burning sensations?

c. By the time you are 12 weeks pregnant, this frequent urination should no longer be a problem, but it is likely to return toward the end of your pregnancy. As the uterus grows, it presses on the urinary bladder, causing the increased frequency of urination during the first trimester. This complaint lessens during the second trimester only to reappear in the third trimester as the fetus begins to descend into the pelvis, causing pressure on the bladder.

During a vaginal exam, the nurse notes that the cervix has a bluish color. The nurse documents this finding as: a. Hegar's sign b. Goodell's sign c. Chadwick's sign d. Ortolani's sign

c. Chadwick's sign Bluish coloration of the cervix is termed Chadwick's sign. Hegar's sign refers to the softening of the lower uterine segment or isthmus. Goodell's sign refers to the softening of the cervix. Ortolani's sign is maneuver done to identify developmental dysplasia of the hip in infants.

When describing amniotic fluid to a pregnant woman, the nurse would include which of the following? a. This fluid acts as the transport mechanism for oxygen and nutrients. b. The fluid is mostly protein to provide nourishment to your baby. c. This fluid acts as a cushion to help to protect your baby from injury d. The amount of fluid remains fairly constant throughout the pregnancy.

c. This fluid acts as a cushion to help to protect your baby from injury Amniotic fluid protects the floating embryo and cushions the fetus from trauma. The placenta acts as a transport mechanism for oxygen and nutrients. Amniotic fluid is primarily water with some organic matter. Throughout pregnancy, amniotic fluid volume fluctuates.

While talking with a pregnant woman who has undergone genetic testing, the woman informs the nurse that her baby will be born with Down syndrome. The nurse understands that Down syndrome is an example of: a. multifactorial inheritance b. X-linked recessive inheritance c. Trisomy numeric abnormality d. Chromosomal deletion

c. Trisomy numeric abnormality Down syndrome is an example of a chromosomal abnormality involving the number of chromosomes (trisomy numeric abnormality), in particular chromosome 21, in which the individual has three copies of that chromosome. Multifactorial inheritance gives rise to disorders such as cleft lip, congenital heart disease, neural tube defects, and pyloric stenosis. X-linked recessive inheritance is associated with disorders such as hemophilia. Chromosomal deletion is involved with disorders such as cri-du-chat syndrome

After teaching a group of adolescent girls about female reproductive development, the nurse determines that teaching was successful when the girls state that menarche is defined as a womans first: a. Sexual experience b. Full hormonal cycle c. menstrual period d. Sign of breast development

c. menstrual period Menarche is defined as the establishment of menstruation. It does not refer to the woman's first sexual experience, full hormonal cycle, or sign of breast development.

The nurse is teaching a pregnant woman about recommended weight gain. The woman has a prepregnancy body mass index of 26. The nurse determines that the teaching was successful when the woman states that she should gain no more than which amount during pregnancy? a. 35 to 40 lbs b. 25 to 34 lbs c. 28 to 40 lbs d. 15 to 25 lbs

d. 15 to 25 lbs A woman with a body mass index of 26 is considered overweight and should gain no more than 15 to 25 lb during pregnancy. Women with a body mass index of 18.5 to 24.9 should gain 25 to 35 lb. A woman with a body mass index less than 18.5 should gain 28 to 40 lb.

The nurse is explaining the events that lead up to ovulation. Which hormone would the nurse identify as being primarily responsible for ovulation? a. Estrogen b. Progesterone c. Follicle-stimulating hormone d. Luteinizing hormone

d. Luteinizing hormone At ovulation, a mature follicle ruptures in response to a surge of luteinizing hormone. Estrogen is predominant at the end of the follicular phase, directly preceding ovulation. Progesterone peaks 5 to 7 days after ovulation. Follicle-stimulating hormone is highest during the first week of the follicular phase of the cycle.

After teaching a group of pregnant women about breast-feeding, the nurse determines that the teaching was successful when the group identifies which hormone as important for the production of breast milk after childbirth? a. Placental estrogen b. Progesterone c. Gonadotropin-releasing hormone d. Prolactin

d. Prolactin After childbirth and expulsion of the placenta, prolactin stimulates the production of milk. Placental estrogen and progesterone stimulate the development of the mammary glands during pregnancy. Gonadotropin-releasing hormone induces the release of follicle-stimulating hormone and luteinizing hormone to assist with ovulation.

A nursing instructor is describing the various childbirth methods. Which of the following would the instructor include as part of the Lamaze method? a. Focus on the pleasurable sensations of childbirth. b. Concentration on sensations while turning on to own bodies c. Interruption of the fear-tension-pain cycle d. use of specific breathing and relaxation techniques

d. use of specific breathing and relaxation techniques Lamaze method: Psychoprophylactic method; mind prevention; use of specific breathing and relaxation techniques

The pill may not be good for someone teenagers because they are often

scatter brained or forgetful.

A group of students is reviewing historical aspects about childbirth. The students demonstrate understanding of the information when they identify the use of twilight sleep as a key event during which time frame?

A) 1700s B) 1800s C) 1900s D) 2000s Ans: C Feedback: Twilight sleep, or the use of a heavy dose of narcotics and amnesiacs, was used on women during childbirth in the United States during the 1900s. Female midwives attended the majority of births during the 1700s. Care shifted to doctors among middle-class women during the 1800s. Certified nurse midwives and childbirth classes characterize the 2000s.

After assessing a woman who has come to the clinic, the nurse suspects that the woman is experiencing dysfunctional uterine bleeding. Which statement by the client would support the nurse's suspicions?

A) "I've been having bleeding off and on that's irregular and sometimes heavy." B) "I get sharp pain in my lower abdomen usually starting soon after my period comes." C) "I get really irritable and moody about a week before my period." D) "My periods have been unusually long and heavy lately." Ans: A Feedback: Dysfunctional uterine bleeding is defined as irregular, abnormal bleeding that occurs with no identifiable anatomic pathology. It is frequently associated with anovulatory cycles, which are common for the first year after menarche and later in life as a woman approaches menopause. Pain occurring with menses refers to dysmenorrhea. Although mood swings may be associated with dysfunctional uterine bleeding, irritability and mood swings are more commonly associated with premenstrual syndrome. Unusually long and heavy periods reflect menorrhagia.

During class, a nursing student asks, "I read an article that was talking about integrative medicine. What is that?" Which response by the instructor would be most appropriate?

A) "It refers to the use of complementary and alternative medicine in place of traditional therapies for a condition." B) "It means that complementary and alternative medicine is used together with conventional therapies to reduce pain or discomfort." C) "It means that mainstream medical therapies and complementary and alternative therapies are combined based on scientific evidence for being effective." D) "It refers to situations in which a client and his or her family prefer to use an unproven method of treatment over a proven one." Ans: C Feedback: Integrative medicine combines mainstream medical therapies and CAM therapies for which there is some scientific evidence of safety and effectiveness (NCCAM, 2011). These include acupuncture, reflexology, therapeutic touch, meditation, yoga, herbal therapies, nutritional supplements, homeopathy, naturopathic medicine, and many more used for the promotion of health and well-being. Complementary medicine is used together with conventional medicine, such as using aromatherapy to reduce discomfort after surgery or to reduce pain during a procedure or during early labor. Alternative medicine is used in place of conventional medicine, such as eating a special natural diet to control nausea and vomiting or to treat cancer instead of undergoing surgery, chemotherapy, or radiation that has been recommended by a conventional doctor.

Parents are complaining about the amount of time their 14-year-old girl spends on the Internet. Which of the following would be most important to address with the parents?

A) "Limit her use of the Internet to an hour per day." B) "Does she do her homework and socialize in person?" C) "Can you place the computer where you can keep an eye on her?" D) "You need to warn her about protecting her identity." Ans: B Feedback: Although having the computer in a family area is better than putting it in her room, and warning her about protecting her identity is key, it is important to determine if the child is neglecting responsibilities, schoolwork, household activities, friends, or other forms of personal interaction. Once this is determined, then reasonable time limits for the child's use of the Internet can be established.

A mother confides to the nurse that she is thinking of divorce. Which suggestion by the nurse would be most helpful in minimizing the effects of the divorce on the couple's son?

A) "Tell him together, using appropriate terms." B) "Reassure him that no one loves him more than you." C) "Do special things with him to make up for the divorce." D) "Share your feelings about your spouse with the child." Ans: A Feedback: Both parents should tell the child about the divorce together, using appropriate terms. The other responses are unacceptable behaviors for the mother, such as competing with the spouse and using the child as a confidant.

The nurse is teaching discipline strategies to the parents of a 4-year-old boy. Which response by the parents indicates a need for more teaching?

A) "We should remove temptations that lead to bad behavior." B) "We need to explain how we expect him to behave." C) "We should tell him we get angry when he's bad." D) "We must praise the child for good behavior." Ans: C Feedback: Telling the child that the parents get angry when he is bad would be inappropriate. This response indicates the need to restate that it is important to let the child know that it is not him but rather his behavior that is bad. Removing temptations, setting expectations, and praising good behavior are important concepts the parents need to learn.

When developing a teaching plan for a couple considering contraception options, which of the following statements would the nurse include?

A) "You should select one that is considered to be 100% effective." B) "The best one is the one that is the least expensive and most convenient." C) "A good contraceptive doesn't require a physician's prescription." D) "The best contraceptive is one that you will use correctly and consistently." Ans: D Feedback: For a contraceptive to be most effective, the client must be able to use it correctly and consistently. Even if a method is considered 100% effective, it is not the best choice if the couple does not use it correctly or consistently. Cost is a consideration, but the least expensive method is not necessarily the best choice. The need for a prescription is not relevant to the couple's choice.

A nursing instructor is describing trends in maternal and newborn health care and the rise in community-based care for childbearing women. The instructor addresses the length of stay for vaginal births during the past decade, citing that which of the following denotes the average stay?

A) 24-48 hours or less B) 72-96 hours or less C) 48-72 hours or less D) 96-120 hours or less Ans: A Feedback: Hospital stays for vaginal births have averaged 24-48 hours or less during the past decade, and 72-96 hours or less for cesarean births.

The nurse is reviewing the medical records of several clients. Which client would the nurse expect to have an increased risk for developing osteoporosis?

A) A woman of African American descent B) A woman who plays tennis twice a week C) A thin woman with small bones D) A woman who drinks one cup of coffee a day Ans: C Feedback: A woman with a small frame and thin bones is at a higher risk for osteoporosis. Caucasian or Asian women, not African American women, are at higher risk for the condition. A woman who plays tennis twice a week is active and thus would be at low risk for osteoporosis. Women who ingest excessive amounts of caffeine are at increased risk.

When caring for childbearing families from cultures different from one's own, which of the following must the nurse accomplish first?

A) Adapting to the practices of the family's culture B) Determining similarities between both cultures C) Assessing personal feelings about that culture D) Learning as much as possible about that culture Ans: C Feedback: The first step is to develop cultural awareness, engaging in self-exploration beyond one's own culture, seeing patients from different cultures, and examining personal biases and prejudices toward other cultures. Once this occurs, the nurse can learn as much about the culture as possible and become familiar with similarities and differences between his or her own culture and the family's culture. The nurse would adapt nursing care to address the practices of the family's culture to provide culturally competent care.

A woman comes to the clinic because she has been unable to conceive. When reviewing the woman's history, which of the following would the nurse least likely identify as a possible risk factor?

A) Age of 25 years B) History of smoking C) Diabetes since age 15 years D) Weight below standard for height and age Ans: A Feedback: Female risk factors for infertility include age older than 27 years, smoking and alcohol consumption, history of chronic illness such as diabetes, and overweight or underweight, which can disrupt hormonal function.

A pregnant woman asks the nurse about giving birth in a birthing center. She says, "I'm thinking about using one but I'm not sure." Which of the following would the nurse need to integrate into the explanation about this birth setting? Select all that apply.

A) An alternative for women who are uncomfortable with a home birth B) The longer length of stay needed when compared to hospital births C) Focus on supporting women through labor instead of managing labor D) View of labor and birth as a normal process requiring no intervention E) Care provided primarily by obstetricians with midwives as backup care Ans: A, C, D Feedback: Birthing centers are an alternative for women who are uncomfortable with a home birth but do not want a hospital birth. A birthing center offers a home-like setting but with close proximity to a hospital in case of complications. Typically the normal discharge time in birthing centers ranges from 4 to 24 hours, shorter than that for a hospital birth. Labor and birth are viewed as a normal process and midwives, not obstetricians, support the woman through labor rather than manage labor. Most centers use a noninterventional view of labor and birth.

A nurse is describing the criteria needed for the diagnosis of premenstrual dysphoric disorder (PMDD). Which of the following would the nurse include as a mandatory requirement for the diagnosis?

A) Appetite changes B) Sleep difficulties C) Persistent anger D) Chronic fatigue Ans: C Feedback: For the diagnosis of PMDD, the woman must exhibit one or more of the following: affective lability such as sadness, tearfulness, or irritability; anxiety and tension; persistent or marked anger or irritability; and depressed mood and feelings of hopelessness. Other symptoms, although not mandatory for the diagnosis, include increased or decreased appetite, sleep difficulties, chronic fatigue, headache, increased or decreased sexual desire, constipation or diarrhea, and breast swelling and tenderness.

A 3-year-old boy with encephalitis is scheduled for a lumbar puncture. Which of the following actions by the nurse would demonstrate atraumatic care?

A) Applying an anesthetic cream before the lumbar puncture B) Having his anxious mother stay in the waiting room C) Explaining, using medical terms, what will happen D) Starting the child's intravenous infusion in his room Ans: A Feedback: Using an anesthetic cream prior to the lumbar puncture reduces the pain associated with the procedure and is an example of atraumatic care. The presence of a parent during procedures is supportive for the child and should be encouraged because it can reduce stress. The explanation of what will happen should be given on the child's level. The IV should not be started in the child's hospital room, which should remain a "safe" area.

The nurse is providing home care for a 6-year-old girl with multiple medical challenges. Which of the following activities would be considered the tertiary level of prevention?

A) Arranging for a physical therapy session B) Teaching the parents to administer albuterol C) Reminding the parents to give the full course of antibiotics D) Giving the DTaP vaccination at proper intervals Ans: A Feedback: The tertiary level of prevention involves restorative, rehabilitative, or quality-of-life care, such as arranging for a physical therapy session. Teaching the parents to administer albuterol and reminding them to give the full course of antibiotics as prescribed are part of the secondary level of prevention, which focuses on diagnosis and treatment of illness. Giving a DTaP vaccination at proper intervals is an example of the primary level of prevention, which centers on health promotion and illness prevention.

A preschool child is scheduled to undergo a diagnostic test. Which action by the nurse would violate a child's bill of health care rights?

A) Arranging for her mother to be with her B) Telling the child the test will not hurt C) Assuring the child that the test will be done quickly D) Introducing the child to the lab technicians Ans: B Feedback: Telling the child the test will not hurt lacks veracity or truth. It is not a lie, but it does not honor the child's right to be educated honestly about his or her health care. Arranging for the mother to be with the child, assuring the child that the test will be done quickly, and introducing the child to the lab technicians are actions that honor the child's bill of health care rights.

The nurse is educating the parents of a 7-year-old girl who has just been diagnosed with epilepsy. Which of the following teaching techniques would be most appropriate?

A) Assessing the parents' knowledge of anticonvulsant medications B) Demonstrating proper seizure safety procedures C) Discussing the surgical procedures for epilepsy D) Giving the parents information in small amounts at a time Ans: D Feedback: Parents, when given a life-altering diagnosis, need time to absorb information and to ask questions. Therefore, giving the parents information in small amounts at a time is best. The child has just been diagnosed with epilepsy, and surgical intervention is not used unless seizures persist despite medication therapy. Therefore, discussing surgery would be inappropriate at this time. Assessing the parents' knowledge of anticonvulsant medications identifies a knowledge gap and need to learn, but it would be unreasonable to think that they would understand the medications because the diagnosis had just been made. Demonstrating proper seizure safety procedures is an effective way to present information later on, once the family begins to understand the condition.

A mother of four children is being interviewed by the nurse. She states: "Whatever my husband and I say goes and the kids need to follow our rules without complaining about them." The nurse interprets this as which parenting style?

A) Authoritative B) Authoritarian C) Permissive D) Rejecting-neglecting Ans: B Feedback: Four major parenting styles seen in our society are authoritarian, authoritative, permissive, and rejecting-neglecting. The authoritarian parent expects obedience from the child and discourages the child from questioning the family's rules. The authoritative or democratic parent shows some respect for the child's opinions. Permissive or laissez-faire parents have little control over the behavior of their children. Rejecting or neglecting parents are indifferent or uninvolved.

After teaching a group of students about premenstrual syndrome, the instructor determines that additional teaching is needed when the students identify which of the following as a prominent assessment finding?

A) Bloating B) Tension C) Dysphoria D) Weight loss Ans: D Feedback: Irritability, fatigue, bloating, tension, and dysphoria are the most prominent and consistently described manifestations of premenstrual syndrome. Weight gain, not weight loss, is associated with this disorder

The nurse would recommend the use of which supplement as a primary prevention strategy to prevent neural tube defects in pregnant women?

A) Calcium B) Folic acid C) Vitamin C D) Iron Ans: B Feedback: Prevention of neural tube defects in the offspring of pregnant women via the use of folic acid is an example of a primary prevention strategy. Calcium, vitamin C, and iron have no effect on the prevention of neural tube defects.

A nurse is assigned to care for an Asian American client. The nurse develops a plan of care with the understanding that based on this client's cultural background, the client most likely views illness as which of the following?

A) Caused by supernatural forces B) A punishment for sins C) Due to spirits or demons D) From an imbalance of forces Ans: D Feedback: Asian Americans believe that illness is caused by an imbalance of forces, specifically yin and yang. Native Americans believe that illness is caused by supernatural forces, and African Americans may believe that illness is caused by spirits or demons.

When integrating the principles of family-centered care for a pregnant woman and her family, the nurse would integrate understanding of which of the following?

A) Childbirth is viewed as a medical event. B) Families are unable to make informed choices. C) Childbirth results in changes in relationships. D) Families require little information to make appropriate decisions. Ans: C Feedback: Family-centered care is based on the following principles. Childbirth affects the entire family and relationships will change. Childbirth is viewed as a normal, healthy event in the life of the family. Families are capable of making decisions about their own care if given adequate information and professional support.

When discussing fetal mortality with a group of students, a nurse addresses maternal factors. Which of the following would the nurse most likely include? Select all that apply.

A) Chromosomal abnormalities B) Malnutrition C) Preterm cervical dilation D) Underlying disease condition E) Poor placental attachment Ans: B Feedback: Fetal mortality may be attributable to maternal factors (e.g., malnutrition, disease, or preterm cervical dilation) or fetal factors (e.g., chromosomal abnormalities or poor placental attachment).

When discussing contraceptive options, which method would the nurse recommend as being the most reliable?

A) Coitus interruptus B) Lactational amenorrheal method (LAM) C) Natural family planning D) Intrauterine system Ans: D Feedback: An intrauterine system is the most reliable method because users have to consciously discontinue using them to become pregnant rather than making a proactive decision to avoid conception. Coitus interruptus, LAM, and natural family planning are behavioral methods of contraception and require active participation of the couple to prevent pregnancy. These behavioral methods must be followed exactly as prescribed.

The nurse is providing care to an ill child and his family. Which of the following activities would deviate from the basic principles of case management?

A) Collaborating with the family throughout the care path B) Focusing on both the client's and the family's needs C) Coordinating care provided by the interdisciplinary team D) Ensuring quality care regardless of the cost Ans: D Feedback: Ensuring quality care regardless of the cost is not part of case management, but providing cost-effective, high-quality care is. Collaborating with the family throughout the care path, coordinating care provided by the interdisciplinary team, and focusing on client and family needs are key components of case management that increase family satisfaction.

After teaching a group of students about the different methods for contraception, the instructor determines that the teaching was successful when the students identify which of the following as a mechanical barrier method? (Select all that apply.)

A) Condom B) Cervical cap C) Cervical sponge D) Diaphragm E) Vaginal ring Ans: A, B, C, D Feedback: Barrier methods include the condom, cervical cap, cervical sponge and diaphragm. The vaginal ring is considered a hormonal method of contraception.

The nurse discusses various contraceptive methods with a client and her partner. Which method would the nurse explain as being available only with a prescription?

A) Condom B) Spermicide C) Diaphragm D) Basal body temperature Ans: C Feedback: The diaphragm is available only by prescription and must be professionally fitted by a health care provider. Condoms and spermicides are available over the counter. Basal body temperature requires the use of a special thermometer that is available over the counter.

A nursing instructor is presenting a class for a group of students about community-based nursing interventions. The instructor determines that additional teaching is needed when the students identify which of the following as a role of the community-based nurse?

A) Conducting childbirth education classes B) Counseling a pregnant teen with anemia C) Consulting with a parent of a child who is vomiting D) Performing epidemiologic investigations Ans: D Feedback: Community health nurses, not community-based nurses, perform epidemiologic investigations to help analyze and develop health policy and community health initiatives. Community-based nurses are involved in teaching, such as childbirth education classes; counseling, such as with a pregnant teenager; and consulting with clients.

When assuming the role of discharge planner for a child requiring ventilator support at home, the nurse would do which of the following?

A) Confer with the school nurse or teacher. B) Teach new self-care skills to the child. C) Determine if there is a need for backup power. D) Discuss coverage with the family's insurance company. Ans: C Feedback: Discharge planning involves the development and implementation of a comprehensive plan for the safe discharge of a client from a health care facility and for continuing safe and effective care in the community and at home. Thus as a discharge planner, the nurse should establish if there is a need for backup power to ensure the safety of the child. Conferring with a school nurse or teacher and dealing with insurance companies are case management activities. Teaching self-care skills is an activity associated with the nurse's role as an educator.

A 9-month-old with glaucoma requires surgery. The infant's parents are divorced. To obtain informed consent, which action would be most appropriate?

A) Contacting the father for informed consent B) Obtaining informed consent from the mother C) Seeking a court ruling on the course of care D) Determining if there is parental sole or joint custody Ans: D Feedback: The most appropriate action would be to determine legal custody by court decree. If the parents have joint custody, then either parent may give consent, but it is always best to have consent given by both parents. The parent with only physical custody may give consent for emergency care. The last resort is getting a court ruling; usually this is not necessary unless the parents disagree about the care of the child.

The nurse who is scheduled to work in a clinic in a Hispanic neighborhood takes time to research Hispanic cultural norms to better provide culturally competent care to people at work. This behavior is an example of which of the following cultural components?

A) Cultural awareness B) Cultural knowledge C) Cultural skills D) Cultural encounter Ans: B Feedback: Cultural knowledge is the acquisition of information about other cultures from a variety of sources. Cultural awareness is an exploration of one's own culture and how values, beliefs, and behaviors have influenced personal life. Cultural skills and practices provide for the incorporation of knowledge of cultural background, including specific practices for health, and a cultural encounter is participation in and interaction with persons of diverse cultural backgrounds.

A 10-year-old girl who is living with a foster family is brought to the clinic for evaluation. When caring for this child, which intervention is a priority?

A) Determining if the child is being bullied at school B) Dealing with mixed expectations of parents C) Establishing who the child's actual caretaker is D) Performing a comprehensive health assessment Ans: D Feedback: Children in foster care are more likely to exhibit a wide range of medical, emotional, behavioral, or developmental problems. Although resilient, they may struggle with certain issues such as unmet health needs. Therefore a comprehensive physical health assessment would be a priority. Determining if the child is being bullied at school is not specific to any one family structure. Assessing for problems related to conflicting feelings toward parents would be more likely common in a blended family. Establishing who the caretaker is would be necessary with a communal family.

The nurse is caring for a 12-year-old child hospitalized for internal injuries following a motor vehicle accident. For which of the following medical treatments would the nurse need to obtain an informed consent beyond the one signed at admission?

A) Diagnostic imaging B) Cardiac monitoring C) Blood testing D) Spinal tap Ans: D Feedback: Most care given in a health care setting is covered by the initial consent for treatment signed when the child becomes a patient at that office or clinic or by the consent to treatment signed upon admission to the hospital or other inpatient facility. Certain procedures, however, require a specific process of informed consent, including major and minor surgery; invasive procedures such as lumbar puncture or bone marrow aspiration; treatments placing the child at higher risk, such as chemotherapy or radiation therapy; procedures or treatments involving research; photography involving children; and applying restraints to children.

A nurse is preparing a class for a group of young adult women about emergency contraceptives (ECs). Which of the following would the nurse need to stress to the group. Select all that apply.

A) ECs induce an abortion like reaction. B) ECs provide some protection against STIs C) ECs are birth control pills in higher, more frequent doses D) ECs are not to be used in place of regular birth control E) ECs provide little protection for future pregnancies. Ans: C, D, E Feedback: Important points to stress concerning ECs are that ECs do not offer any protection against STIs or future pregnancies; should not be used in place of regular birth control, as they are less effective; are regular birth control pills given at higher doses and more frequently; and are contraindicated during pregnancy (Miller, 2011). Contrary to popular belief, ECs do not induce abortion and are not related to mifepristone or RU-486, the so-called abortion pill approved by the FDA in 2000.

The parents of an 8-year-old with cancer are telling the nurse their problems and successes when caring for their child. In response, the nurse arranges for social services to meet with the parents to help them obtain financial assistance for the equipment and supplies. The nurse is acting in which role?

A) Educator B) Advocate C) Case manager D) Direct care provider Ans: B Feedback: The nurse is acting as an advocate, representing the client and family to a third party, by ensuring that the family has the resources and services to provide care for their child. The nurse acts as a direct care provider through assessment, observation of physical care, and the actual provision of physical care. The role of educator would require the nurse to give rather than receive information. Case management involves coordinating elements of a nursing plan of care.

When preparing a teaching plan for a group of first-time pregnant women, the nurse expects to review how maternity care has changed over the years. Which of the following would the nurse include when discussing events of the 20th century?

A) Epidemics of puerperal fever B) First cesarean birth C) X-rays used to assess pelvic size D) Development of freestanding birth centers Ans: D Feedback: In the 20th century (1900s), freestanding birth centers were developed. Puerperal fever epidemics, the first cesarean birth, and the use of x-rays to assess pelvic size were events occurring during the 19th century (1800s).

A nurse is developing cultural competence. Which of the following indicates that the nurse is in the process of developing cultural knowledge? Select all that apply.

A) Examining personal sociocultural heritage B) Reviewing personal biases and prejudices C) Seeking resources to further understanding of other cultures D) Becoming familiar with other culturally diverse lifestyles E) Performing a competent cultural assessment F) Advocating for social justice to eliminate disparities Ans: C, D Feedback: When developing cultural knowledge, the nurse would seek resources to increase understanding of different sociocultural groups and become familiar with culturally/ethnically diverse groups, worldviews, beliefs, practices, lifestyles, and problem-solving styles. Examining one's personal sociocultural heritage and personal biases and prejudices are components involved with developing cultural awareness. Performing a competent cultural assessment and advocating for social justice are components involved with developing cultural skills.

The nurse is caring for a 14-year-old girl with multiple health problems. Which of the following activities would best reflect evidence-based practice by the nurse?

A) Following blood pressure monitoring recommendations B) Determining how often vital signs are monitored C) Using hospital protocol for ordering diagnostic tests D) Deciding on the medication dose Ans: A Feedback: Using hospital protocol for ordering a diagnostic test, determining how often vital signs are monitored, and deciding on the medication dose would be the physician's responsibility. However, following blood pressure monitoring recommendations would be part of evidence-based practice reflected in the nursing care delivered.

The school nurse is trying to get consent to care for an 11-year-old boy with diabetic ketoacidosis. His parents are out of town on vacation and the child is staying with a neighbor. Which action would be the priority?

A) Getting telephone consent, with two people listening to the verbal consent B) Providing emergency care without parental consent C) Contacting the child's aunt or uncle to obtain consent D) Advocating for parens patriae to proceed with care for the child Ans: A Feedback: The priority action would be to contact the neighbor for an emergency number to reach the parents and get their verbal consent, with two witnesses listening simultaneously. If the nurse cannot reach the parents and there is no relative or other person with written authorization to act on the parent's behalf, then the physician may initiate emergency care without the parent's consent. Parens patriae would be reserved for situations in which the parents are neglectful, irresponsible, or incompetent, for example, if the parents refuse treatment and the health care team feels the treatment is reasonable and warranted.

The nurse providing care to a family and an ill child in a hospital setting reflects on the focus of the health care provided in today's society. Which of the following statements best describes the current definition of health?

A) Health is described as "an absence of disease." B) Health is measured by monitoring the mortality and morbidity of a group. C) Health is a state of complete physical, mental, and social well-being. D) Technological gains made in health care are shared equally among all children. Ans: C Feedback: In the past, health was defined simply as the absence of disease; health was measured by monitoring the mortality and morbidity of a group. Over the past century, however, the focus of health has shifted to disease prevention, health promotion, and wellness. The World Health Organization (2012) defines health as "a state of complete physical, mental, and social well-being, and not merely the absence of disease or infirmity." The gains in child health have been huge, but, unfortunately, these gains are not shared equally among all children.

After teaching a group of nursing students about pregnancy-related mortality, the instructor determines that additional teaching is necessary when the students identify which condition as a leading cause?

A) Hemorrhage B) Embolism C) Hypertension D) Infection Ans: B Feedback: Most pregnancy-related complications are preventable. The leading causes of pregnancy-related mortality are hemorrhage, infection, preeclampsia-eclampsia, obstructed labor, and unsafe abortion. Embolism is not a leading cause.

Which finding would the nurse expect to find in a client with endometriosis?

A) Hot flashes B) Dysuria C) Fluid retention D) Fever Ans: B Feedback: The client with endometriosis is often asymptomatic, but clinical manifestations include painful urination, pain before and during menstrual periods, pain during or after sexual intercourse, infertility, depression, fatigue, painful bowel movements, chronic pelvic pain, hypermenorrhea, pelvic adhesions, irregular and more frequent menses, and premenstrual spotting. Hot flashes may be associated with premenstrual syndrome or menopause. Fluid retention is associated with premenstrual syndrome. Fever would suggest an infection.

The nurse is making a home visit to a client who had a cesarean birth 3 days ago. Assessment reveals that she is complaining of intermittent pain, rating it as 8 on a scale of 1 to 10. She states, "I'm pretty tired. And with this pain, I haven't been drinking and eating like I should. The medication helps a bit but not much. My mom has been helping with the baby." Her incision is clean, dry, and intact. Which nursing diagnosis would the nurse identify as the priority for this client?

A) Impaired skin integrity related to cesarean birth incision B) Fatigue related to effects of surgery and caretaking activities C) Imbalanced nutrition, less than body requirements, related to poor fluid and food intake D) Acute pain related to incision and cesarean birth Ans: D Feedback: The client reports a pain rating of 8 out of 10 and states that the medication is helping only a bit. She also mentions that the pain is interfering with her ability to eat and drink. Therefore, the priority nursing diagnosis is acute pain related to incision and cesarean birth. Her incision is clean, dry, and intact, so impaired skin integrity is not the problem. She is fatigued, but her complaints of pain supersede her fatigue. Although her nutritional intake is reduced, it is due to the pain.

The student nurse is learning about the past history of child health and health care in the United States. Which of the following statements by the student indicates successful learning about the condition of health care in the past and current centuries?

A) In past centuries in the United States, the health of the country was better than it is today due to the simpler style of living. B) In the current century, mortality rates are high, but life expectancy has increased due to technological advances. C) In the late 18th and early 19th centuries urban public health improvements made cities healthier places for growing children. D) By the end of the 20th century, unintentional injuries rather than infectious diseases had become the leading cause of death for children older than 1 year old. Ans: D Feedback: In past centuries in the United States, the health of the country was poorer than it is today; mortality rates were high and life expectancy was short. Over the years the health of children received more and more attention, leading to a better understanding of sources of illness and improvements in sanitation, water, and nutrition. As a result, by the end of the late 20th century, unintentional injuries rather than infectious diseases had become the leading cause of death for children older than 1 year old.

After teaching a group of students about the changes in health care delivery and funding, which of the following, if identified by the group as a current trend seen in the maternal and child health care settings, would indicate that the teaching was successful?

A) Increase in ambulatory care B) Decrease in family poverty level C) Increase in hospitalization of children D) Decrease in managed care Ans: A Feedback: The health care system has moved from reactive treatment strategies in hospitals to a proactive approach in the community, resulting in an increased emphasis on health promotion and illness prevention in the community through the use of community-based settings such as ambulatory care. Poverty levels have not decreased and the hospitalization of children has not increased. Case management also is a primary focus of care.

When explaining community-based nursing versus nursing in the acute care setting to a group of nursing students, the nurse describes the challenges associated with community-based nursing. Which of the following would the nurse include?

A) Increased time available for education B) Improved access to resources C) Decision making in isolation D) Greater environmental structure Ans: C Feedback: Community-based nurses often have to make decisions in isolation. This is in contrast to the acute care setting, where other health care professionals are readily available. Nursing care and procedures in the community also are becoming more complex and time-consuming, leaving limited time for education. Nurses working in the community have fewer resources available and the environment is less structured and controlled when compared to the acute care setting.

The nurse is performing a health assessment of a school-age child. Based on the child's developmental level, on which of the following problems would the nurse focus more attention?

A) Infections B) Poisonings C) Risk-taking behaviors D) Accidents and injuries Ans: D Feedback: Developmental level has a major impact on the health status of children. The physiologic immaturity of an infant's body systems increases the risk for infection. Ingestion of toxic substances and risk of poisoning are major health concerns for toddlers as they become more mobile and inquisitive. Because preschool- and school-age children are, generally, very active, they are more prone to injury and accidents. Adolescents are establishing their identity, which may lead them to separate from the family values and traditions for a period of time and attempt to conform to their peers. This journey may lead to risk-taking behaviors, resulting in injuries or other situations that may impair their health.

The nurse is counseling a young couple who in 2 months are having their third baby. The nurse uses Von Bertalanffy's (1968) general system theory applied to families to analyze the family structure. Which of the following best describes the main emphasis of this theory and its application to family dynamics?

A) It emphasizes the family as a system with interdependent, interacting parts that endure over time to ensure the survival, continuity, and growth of its components. B) It emphasizes the social system of family, such as the organization or structure of the family and how the structure relates to the function. C) It emphasizes the developmental stages that all families go through, beginning with marriage; the longitudinal career of the family is also known as the family life cycle. D) It addresses the way families respond to stress and how the family copes with the stress as a group and how each individual member copes. Ans: A Feedback: Von Bertalanffy emphasizes the family as a system with interdependent, interacting parts that endure over time to ensure the survival, continuity, and growth of its components. Friedman emphasizes the social system of family, such as the organization or structure of the family and how the structure relates to the function. Duvall emphasizes the developmental stages that all families go through, beginning with marriage; the longitudinal career of the family is also known as the family life cycle. The family stress theory addresses the way families respond to stress and how the family copes with the stress as a group and how each individual member copes.

When reviewing the medical record of a client diagnosed with endometriosis, which of the following would the nurse identify as a risk factor for this woman?

A) Low fat in the diet B) Age of 14 years for menarche C) Menstrual cycles of 24 days D) Short menstrual flow Ans: C Feedback: Risk factors for developing endometriosis include increasing age, family history of endometriosis in a first-degree relative, short menstrual cycle (less than 28 days), long menstrual flow (more than 1 week), high dietary fat consumption, young age at menarche (younger than age 12), and few (one or two) or no pregnancies.

The nurse caring for families in crisis assesses the affective function of an immigrant family consisting of a father, mother, and two school-age children. Based on Friedman's structural functional theory, which of the following would the nurse identify as defining the family component?

A) Meeting the love and belonging needs of each member B) Teaching children how to function and assume adult roles in society C) Ensuring the family has necessary resources with appropriate allocation D) Involving the provision of physical care to keep the family healthy Ans: A Feedback: According to Friedman's structural functional theory, the affective function involves meeting the love and belonging needs of each member. Teaching children how to function and assume adult roles in society is the socialization and social placement function. Ensuring the family has necessary resources with appropriate allocation is related to the economic function, and involving the provision of physical care to keep the family healthy involves the health care function.

The nurse is assessing a toddler for temperament and documents a "difficult" temperament. Which of the following is a trait of this type of personality?

A) Moodiness B) Irritability C) Even-temperedness D) Overly active Ans: D Feedback: Children's temperaments may be categorized into three major groups: easy, difficult, and slow to warm up. Easy children are even-tempered and have regular biological functions, predictable behavior, and a positive attitude toward new experiences. Difficult children are irritable, overactive, and intense; they react to new experiences by withdrawing and are frustrated easily. Children in the slow-to-warm-up category are moody and less active and have more irregular reactions; they react to new experiences with mild but passive resistance and need extra time to adjust to new situations.

The nurse is reviewing the laboratory test results of a client with dysfunctional uterine bleeding (DUB). Which finding would be of concern?

A) Negative pregnancy test B) Hemoglobin level of 10.1 g/dL C) Prothrombin time of 60 seconds D) Serum cholesterol of 140 mg/dL Ans: B Feedback: A hemoglobin level of 10.1 g/dL suggests anemia, which might occur secondary to prolonged or heavy menses. A negative pregnancy test, a prothrombin time of 60 seconds, and a serum cholesterol level of 140 mg/dL are within normal parameters.

The neonatal nurse researches the neonatal mortality rate in the United States. Which of the following accurately describes this measurement of child health? Select all that apply.

A) Neonatal mortality is the number of infant deaths occurring in the first 28 days of life per 1,000 live births. B) The infant mortality rate refers to the number of deaths occurring in the first 6 months of life. C) Neonatal mortality is documented as the number of deaths in relation to 1,000 live births. D) The infant mortality rate is used as an index of the general health of a country. E) In 2010, the U.S. infant mortality rate for White infants was greater than that for Black infants per 1,000 live births. Ans: A Feedback: Neonatal mortality is the number of infant deaths occurring in the first 28 days of life per 1,000 live births. The infant mortality rate refers to the number of deaths occurring in the first 12 months of life and is documented as the number of deaths in relation to 1,000 live births. The infant mortality rate is used as an index of the general health of a country; generally, this statistic is one of the most significant measures of children's health. In 2010, the infant mortality rate in the United States was 6.14 for White infants and 11.61 for Black infants per 1,000 live births. In 2005, the infant mortality rate in the United States was 6.86 per 1,000 live births.

A woman has opted to use the basal body temperature method for contraception. The nurse instructs the client that a rise in basal body temperature indicates which of the following?

A) Onset of menses B) Ovulation C) Pregnancy D) Safe period for intercourse Ans: B Feedback: Basal body temperatures typically rise within a day or two after ovulation and remain elevated for approximately 2 weeks, at which point bleeding usually begins. Basal body temperature is not a means for determining pregnancy. Having intercourse while the temperature is elevated would increase the risk of pregnancy.

After discussing various methods of contraception with a client and her partner, the nurse determines that the teaching was successful when they identify which contraceptive method as providing protection against sexually transmitted infections (STIs)?

A) Oral contraceptives B) Tubal ligation C) Condoms D) Intrauterine system Ans: C Feedback: Condoms are a barrier method of contraception. In addition to providing a physical barrier for sperm, they also protect against STIs. Oral contraceptives, tubal ligation, and intrauterine systems provide no protection against STIs.

The nurse working in a free community clinic knows that access to health care is affected negatively by lack of health insurance. Which of the following accurately describe the effect of lack of insurance on family health? Select all that apply.

A) Parents with uninsured children often delay care and are less likely to take their children to a doctor or dentist for preventive care. B) The percentage of children without health insurance is beginning to decrease slightly from previous years. C) Currently, the states have improved enrollment in Medicaid and the State Children's Health Insurance Program (SCHIP). D) Despite state efforts to ensure all children, nearly two-thirds of children eligible for these public programs are not enrolled. E) In most states, men and women of the same health status and age are charged similar rates for the same individual health insurance policy. Ans: A, C, D Feedback: Parents with uninsured children often delay care for their children, and are less likely to take their children to a doctor or dentist for preventive care. The percentage of children without health insurance is rising. Despite efforts by the states to improve enrollment in Medicaid and State Children's Health Insurance Program (SCHIP), nearly two-thirds of children eligible for these programs remain uninsured. Additionally, in most states, a man and woman of the same age and health status will be charged different rates for exactly the same individual health insurance policy.

A client comes to the clinic with abdominal pain. Based on her history the nurse suspects endometriosis. The nurse expects to prepare the client for which of the following to confirm this suspicion?

A) Pelvic examination B) Transvaginal ultrasound C) Laparoscopy D) Hysterosalpingogram Ans: C Feedback: The only certain method of diagnosing endometriosis is by seeing it. Therefore, the nurse would expect to prepare the client for a laparoscopy to confirm the diagnosis. A pelvic examination and transvaginal ultrasound are done to assess for endometriosis but do not confirm its presence. Hysterosalpingography aids in identifying tubal problems resulting in infertility.

When teaching a group of postmenopausal women about hot flashes and night sweats, the nurse would address which of the following as the primary cause?

A) Poor dietary intake B) Estrogen deficiency C) Active lifestyle D) Changes in vaginal pH Ans: B Feedback: Hot flashes and night sweats are classic signs of estrogen deficiency. They are unrelated to dietary intake or active lifestyle. Changes in vaginal pH are associated with genitourinary changes of menopause.

A client who has come to the clinic is diagnosed with endometriosis. Which of the following would the nurse expect the physician to prescribe as a first-line treatment?

A) Progestins B) Antiestrogens C) Gonadotropin-releasing hormone analogues D) NSAIDs Ans: A Feedback: Although progestins, antiestrogens, and gonadotrophin-releasing analogues are used as treatment options for endometriosis, NSAIDS are considered the first-line treatment to reduce pain.

A nurse is preparing a class for a group of women at a family planning clinic about contraceptives. When describing the health benefits of oral contraceptives, which of the following would the nurse most likely include? (Select all that apply.)

A) Protection against pelvic inflammatory disease B) Reduced risk for endometrial cancer C) Decreased risk for depression D) Reduced risk for migraine headaches E) Improvement in acne Ans: A, B, E Feedback: The health benefits of oral contraceptives include protection against pelvic inflammatory disease, a reduced risk for endometrial cancer, and improvement in acne. Oral contraceptives are associated with an increased risk for depression and migraine headaches.

Which action would the nurse include in a primary prevention program in the community to help reduce the incidence of HIV infection?

A) Provide treatment for clients who test positive for HIV. B) Monitor viral load counts periodically. C) Educate clients about how to practice safe sex. D) Offer testing for clients who practice unsafe sex. Ans: C Feedback: Primary prevention involves preventing disease before it occurs. Therefore, educating clients about safe-sex practices would be an example of a primary prevention strategy. Providing treatment for clients who test positive for HIV, monitoring viral loads periodically, and offering testing for clients who practice unprotected sex are examples of secondary preventive strategies, which focus on early detection and treatment of adverse health conditions.

The United States ranks 50th in the world for maternal mortality and 41st among industrialized nations for infant mortality rates. When developing programs to assist in decreasing theses rates, which factor would most likely need to be addressed as having the greatest impact?

A) Resolving all language and cultural differences B) Ensuring early and adequate prenatal care C) Providing more extensive women's shelters D) Encouraging all women to eat a balanced diet Ans: B Feedback: The lack of prenatal care during pregnancy is a major factor contributing to a poor outcome. Prenatal care is well known to prevent complications of pregnancy and to support the birth of healthy infants. Infant mortality commonly includes problems occurring at birth or shortly thereafter. Thus, ensuring early and adequate prenatal care would have the greatest impact on decreasing these rates. Resolving all language and cultural differences would be helpful but is unrealistic. Providing more extensive women's shelters would be helpful for women who are victims of abuse. Encouraging all women to eat a balanced diet is helpful but would not decrease infant mortality rates.

A nursing student is reviewing information about documenting client care and education in the medical record and the purposes that it serves. The student demonstrates a need for additional study when the student identifies which of the following as a reason for documentation?

A) Serves as a communication tool for the interdisciplinary team B) Demonstrates education the family has received if legal matters arise C) Permits others access to allow refusal of medical insurance coverage D) Verifies meeting client education standards set by the Joint Commission Ans: C Feedback: Medical records are not in place for others to view for the sole purpose of denying medical coverage. Documenting client care and education (medical records) serves four main purposes. The client's medical record serves as a communication tool that the entire interdisciplinary team can use to keep track of what the client and family have learned. Next, it serves to testify to the education the family has received if legal matters arise. Third, it verifies standards set by JCAHO and other accrediting bodies that hold health care providers accountable for client education activities. Last, it informs third-party payers of goods and services provided for reimbursement purposes.

The nurse is working with a group of community leaders to develop a plan to address the special health needs of women. Which of the following conditions would the group address as the major problem?

A) Smoking B) Heart disease C) Diabetes D) Cancer Ans: B Feedback: The group needs to address cardiovascular disease, the number-one cause of death in women regardless of racial or ethnic group. Smoking is related to heart disease and cancer, although heart disease and cancer can occur in any woman regardless of her smoking history. Cancer is the second leading cause of death, with women having a one in three lifetime risk of developing cancer. Diabetes is another important health condition that can affect women, but it is not the major health problem that heart disease is.

The nurse is caring for a 2-week-old girl with a metabolic disorder. Which of the following activities would deviate from the characteristics of family-centered care?

A) Softening unpleasant information or prognoses B) Evaluating and changing the nursing plan of care C) Collaborating with the child and family as equals D) Showing respect for the family's beliefs and wishes Ans: A Feedback: Family-centered care requires that the nurse provide open and honest information to the child and family. It is inappropriate to soften unpleasant information or prognoses. Evaluating and changing the nursing plan of care to fit the needs of the child and family, collaborating with them as equals, and showing respect for their beliefs and wishes are guidelines for family-centered care.

Which of the following would the nurse emphasize when teaching postmenopausal women about ways to reduce the risk of osteoporosis?

A) Swimming daily B) Taking vitamin A C) Following a low-fat diet D) Taking calcium supplements Ans: D Feedback: Osteoporosis is a condition in which bone mass declines to such an extent that fractures occur with minimal trauma. Increasing calcium and vitamin D intake is a major preventive measure. Other measures to reduce the risk include engaging in weight-bearing exercise such as walking. Swimming, although a beneficial exercise, is not a weight-bearing exercise. Taking vitamin A supplements would have no effect on preventing bone loss. Following a low-fat diet would be helpful in reducing the risk for cardiovascular disease.

A client is to receive an implantable contraceptive. The nurse describes this contraceptive as containing:

A) Synthetic progestin B) Combined estrogen and progestin C) Concentrated spermicide D) Concentrated estrogen Ans: A Feedback: Implantable contraceptives deliver synthetic progestin that acts by inhibiting ovulation and thickening cervical mucus so sperm cannot penetrate. Implantable contraceptives do not contain combined estrogen and progestin, concentrated spermicide, or concentrated estrogen.

Which of the following measures would the nurse include in the teaching plan for a woman to reduce the risk of osteoporosis after menopause?

A) Taking vitamin supplements B) Eating high-fiber, high-calorie foods C) Restricting fluid to 1,000 mL daily D) Participating in regular daily exercise Ans: D Feedback: Measures to reduce osteoporosis after menopause include daily weight-bearing exercise, increasing calcium and vitamin D intake, and avoiding smoking and excessive alcohol intake. General vitamin supplements may be helpful overall, but they are not specific to reducing the risk of osteoporosis. A diet high in calcium and vitamin D, not fiber and calories, would be appropriate. Restricting fluids would have no effect on preventing osteoporosis.

A nurse is preparing a teaching plan for a woman who is pregnant for the first time. Which of the following would the nurse incorporate into the teaching plan to foster the client's learning? Select all that apply.

A) Teach "survival skills" first. B) Use simple, nonmedical language. C) Refrain from using a hands-on approach. D) Avoid repeating information. E) Use visual materials such as photos and videos. Ans: A, B, E Feedback: To foster learning, the nurse should slow down and repeat information often; speak in a conversational style using plain, nonmedical language; prioritize information, teaching "survival skills" first; use visuals; and use an interactive, "hands-on" approach.

After interviewing the mother of an 8-year-old girl with a broken arm, the nurse identifies the mother as the nurturer in the family. When developing the teaching plan for the girl and her mother, the nurse integrates knowledge of this role and focuses the teaching on which of the following?

A) Teaching proper care procedures B) Dealing with insurance coverage C) Determining success of treatment D) Transmitting information to family members Ans: A Feedback: The mother is the nurturer and thus the primary caregiver, so she would be the best family member to learn proper care procedures. Dealing with insurance coverage is the responsibility of the family's financial manager, transmitting information to family members is the gatekeeper's duty, and determining the success of treatment would likely fall to the family decision maker.

After teaching a group of students about the different levels of prevention, the instructor determines a need for additional teaching when the students identify which of the following as an activity at the tertiary prevention level in community-based health care?

A) Teaching women to take folic acid supplements to prevent neural tube defects B) Working with women who are victims of domestic violence C) Working with clients at an HIV clinic to provide nutritional and CAM therapies D) Teaching hypertensive clients to monitor blood pressure Ans: A Feedback: Primary prevention involves preventing a disease or condition before it occurs, such as teaching women about the importance of taking folic acid supplements to prevent neural tube defects. Secondary prevention is the early detection of disease or conditions. Working with women who are victims of domestic violence, clients at an HIV clinic, or hypertensive clients are all examples of tertiary prevention, which is designed to reduce or limit the progression of a permanent, irreversible disease or disability.

The nurse is caring for a 4-year-old boy with Ewing's sarcoma who is scheduled for a CT scan tomorrow. Which of the following best reflects therapeutic communication?

A) Telling him he will get a shot when he wakes up tomorrow morning B) Telling him how cool he looks in his baseball cap and pajamas C) Using family-familiar words and soft words when possible D) Describing what it is like to get a CT scan using words he understands Ans: D Feedback: Describing what it is like to get a CT scan using age-appropriate words is the best example of therapeutic communication. It is goal-directed, focused, and purposeful communication. Using family-familiar words and soft words is a good teaching technique. Telling him how cool he looks in his baseball cap and pajamas is not goal-directed communication. Telling the child he will get a shot when he wakes up could keep him awake all night.

After describing the procedure and medical necessity, the nurse asks a 14-year-old child to assent to a skin graft. Which of the following statements accurately describes the requirements for this type of assent?

A) The age of assent occurs at 12 years old. B) It is not necessary to obtain assent from a minor for a procedure. C) A minor can dissent to a procedure but his or her wishes are not binding. D) In some cases, such as cases of significant morbidity or mortality, dissent may need to be overridden. Ans: D Feedback: Assent means agreeing to something. In pediatric health care, the term assent refers to the child's participation in the decision-making process about health care (McCullough & Stein, 2010). In some cases, such as cases of significant morbidity or mortality, dissent may need to be overridden. The age of assent depends on the child's developmental level, maturity, and psychological state. The converse of assent, dissent (disagreeing with the treatment plan), when given by an adolescent 13 to 17 years of age, is considered binding in some states. The American Academy of Pediatrics proposes that a child with an intellectual age of 7 years or older is competent to understand participation in research and therefore provide assent (Sirbaugh & Diekema, 2011).

The nurse is teaching discipline strategies to the parents of a 12-year-old girl. Which of the following topics is an example of positive reinforcement discipline?

A) Unplugging the DVD player for the weekend B) Taking a chore away from the child for a week C) Having her clean up the kitchen for a week D) Ignoring her request if she doesn't say "please" Ans: B Feedback: Taking a chore away from the child for a week is an excellent way to reward her for positive behavior. Unplugging the DVD player and assigning an extra chore are examples of negative reinforcement. Ignoring her until she uses good manners is an example of extinction.

A nurse is considering a change in employment from the acute care setting to community-based nursing. The nurse is focusing her job search on ambulatory care settings. Which of the following would the nurse most likely find as a possible setting? Select all that apply.

A) Urgent care center B) Hospice care C) Immunization clinic D) Physician's office E) Day surgery center F) Nursing home Ans: A, D, E Feedback: Ambulatory care settings include physician's offices, day surgery centers, and freestanding urgent care centers. Hospice may be a part of home health care services or long-term care. Immunization clinics are examples of health department services. Nursing homes are examples of long-term care settings.

A nurse is working to develop a health education program for a local community to address breast cancer awareness. Which of the following would the nurse expect to include when describing this problem to the group? Select all that apply.

A) White women have higher rates of breast cancer than African American women. B) African American women are more likely to die from breast cancer at any age. C) Survival at any stage is worse among White women. D) Women living in South America have the highest rates of breast cancer. E) Breast cancer is the leading cause of cancer mortality in women. Ans: A Feedback: White women get breast cancer at a higher rate than African American women, but African American women are more likely to get breast cancer before they are 40, and are more likely to die from it at any age. This is likely because the cancer is more advanced when it is found in African American women, and because survival at every cancer stage is worse among African American women. Women living in North America have the highest rate of breast cancer in the world. At this time there are about 2.6 million breast cancer survivors in the United States. It is the most common malignancy in women and second only to lung cancer as a cause of cancer mortality in women.

A nurse is preparing a presentation for a local women's group about heart disease and women. Which of the following would the nurse expect to address when discussing measures to promote health?

A) Women have similar symptoms as men for a heart attack. B) Heart disease is no longer viewed as a "man's disease." C) Women experiencing a heart attack are at greater risk for dying. D) Heart attacks in women are more easily diagnosed. Ans: C Feedback: Women who have a heart attack are more likely than men to die. Heart attacks in women are often more difficult to diagnose than in men because of their vague and varied symptoms. Heart disease is still thought of as a "man's disease," and thus a heart attack may not be considered in the differential diagnosis when a woman presents to the emergency room. Nurses need to look beyond the obvious "crushing chest pain" textbook symptom that heralds a heart attack in men. Manifestations of heart disease differ between men and women.

Which suggestion by the nurse about pushing would be most appropriate to a woman in the second stage of labor? A. "Choose whatever method you feel most comfortable with for pushing." B. "Lying flat with your head elevated on two pillows makes pushing easier." C. "Bear down like you're having a bowel movement with every contraction." D. "Let me help you decide when it is time to start pushing."

A. "Choose whatever method you feel most comfortable with for pushing."

A nurse is presenting a discussion on sexual violence at a local community college. When describing the incidence of sexual violence, the nurse would identify that a woman has which chance of experiencing a sexual assault in her lifetime? A. 1 in 5 B. 1 in 3 C. 3 in 20 D. 2 in 15

A. 1 in 5

A client is admitted in the healthcare facility with pelvic inflammatory disease (PID). When reviewing the client's history, which of the following would the nurse identify as a risk factor? A. Frequent douching B. Environmental exposure C. Genetic predisposition D. Gestational diabetes

A. Frequent douching

A woman is at 20 weeks' gestation. The nurse would expect to find the fundus at which area? A. at the level of the umbilicus B. midway between the pubis and umbilicus C. just above the symphysis pubis D. midway between the umbilicus and xiphoid process

A. at the level of the umbilicus

A woman comes to the clinic reporting vaginal discharge. The nurse suspects trichomoniasis based on which symptoms? Select all that apply. A. urinary frequency B. yellow/green discharge C. joint pain D. blister-like lesions E. muscle aches

A. urinary frequency B. yellow/green discharge

A woman in labor is to receive continuous internal electronic fetal monitoring. The nurse prepares the client for this monitoring based on the understanding that which criterion must be present? A. a neonatologist to insert the electrode B. cervical dilation of 2 cm or more C. floating presenting fetal part D. intact membranes

B. cervical dilation of 2 cm or more

A nurse is describing the cycle of violence to a community group. When explaining the first phase, the nurse would most likely include which description? A. somehow triggered by the victim's behavior B. characterized by tension-building and minor battery C. associated with loss of physical and emotional control D. like a honeymoon that lulls the victim

B. characterized by tension-building and minor battery

A nurse is reviewing the results of four clients who have undergone amniocentesis. Which client would the nurse recommend that the health care provider see first? A. client at 34 weeks' gestation with gestational diabetes and L/S ratio of 2:1 B. client at 38 weeks' gestation with fetal heart rate of 110 and green amniotic fluid sample C. client at 36 weeks' gestation with preeclampsia and amniotic fluid negative for bilirubin D. client at 16 weeks' gestation with placenta previa and high alpha-fetoprotein level

B. client at 38 weeks' gestation with fetal heart rate of 110 and green amniotic fluid sample

A nurse is caring for a woman who was recently raped. The nurse would expect this woman to experience which phase first? A. reorganization B. disorganization C. integration D. denial

B. disorganization

A nurse is monitoring a client's hCG levels because she has had a previous ectopic pregnancy and one spontaneous abortion. Which finding would the nurse interpret as indicating that the pregnancy is progressing appropriately? A. plateauing of the level at 7 days B. doubling of the level every 2 to 3 days C. abruptly declining levels after 60 days D. gradually increasing levels every month

B. doubling of the level every 2 to 3 days

A woman has just entered the second stage of labor. The nurse would focus care on which intervention? A. palpating the woman's fundus for position and firmness B. encouraging the woman to push when she has a strong desire to do so C. alleviating perineal discomfort with the application of ice packs D. completing the identification process of the newborn with the mother

B. encouraging the woman to push when she has a strong desire to do so

Which finding would the nurse expect to find in a client with bacterial vaginosis? A. vaginal pH of 3 B. fish-like odor of discharge C. cervical bleeding on contact D. yellowish-green discharge

B. fish-like odor of discharge

During a routine prenatal visit, a client, 36 weeks pregnant, states she has difficulty breathing and feels like her pulse rate is really fast. The nurse finds her pulse to be 100 beats per minute (increased from baseline readings of 70 to 74 beats per minute) and irregular, with bilateral crackles in the lower lung bases. The nurse would develop a plan of care identifying interventions to promote which area as the priority? A. activity B. gas exchange C. anxiety D. tissue perfusion

B. gas exchange

The nurse is developing a plan of care for a client who is receiving highly active antiretroviral therapy (HAART) for treatment of HIV. What would the nurse identify as the goal of this therapy? A. conduct additional drug research B. improve morbidity and mortality rates C. promote the progression of disease D. intervene in late-stage AIDS

B. improve morbidity and mortality rates

A woman telephones her health care provider and reports that her water just broke. Which suggestion by the nurse would be most appropriate? A. "Come in as soon as you feel the urge to push." B. "Call us back when you start having contractions." C. "Come to the clinic or emergency department for an evaluation." D. "Drink 3 to 4 glasses of water and lie down."

C. "Come to the clinic or emergency department for an evaluation."

A pregnant woman in the 36th week of gestation reports that her feet are quite swollen at the end of the day. After careful assessment, the nurse determines that this is an expected finding at this stage of pregnancy. Which intervention is appropriate for the nurse to suggest? A. "Limit your intake of fluids." B. "Eliminate salt from your diet." C. "Try elevating your legs when you sit." D. "Wear spandex-type full-length pants."

C. "Try elevating your legs when you sit."

A nurse is providing care to a woman in labor. The nurse determines that the client is in the active phase based on which assessment findings? Select all that apply. A. cervical effacement of 90% B. strong desire to push C. contractions every 2 to 3 minutes D. contractions every 90 seconds E. cervical dilation of 6 cm

C. contractions every 2 to 3 minutes E. cervical dilation of 6 cm

A nurse is auscultating the chest of a client at 16 weeks' gestation. The nurse immediately notifies the health care provider about which finding? A. soft systolic murmur B. clear breath sounds C. heart rate 25 bpm above baseline D. symmetrical chest movement.

C. heart rate 25 bpm above baseline

When developing a teaching plan for a community group about HIV infection, which group would the nurse identify as an emerging risk group for HIV infection? A. new health care workers B. Asian immigrants C. heterosexual women D. Native Americans

C. heterosexual women

The nurse is teaching a health education class on male reproductive anatomy and asks the students to identify the site of sperm production. Which structure, if identified by the group, would indicate to the nurse that the teaching was successful? a. Testes b. Seminal vesicles c. Scrotum d. Prostate gland

a. Testes The testes are responsible for sperm production. The seminal vesicles produce nutrient seminal fluid. The scrotum surrounds and protects the testes. The prostate gland and the seminal vesicles produce fluid to nourish the sperm

A nurse is describing advances in genetics to a group of students. Which of the following would the nurse least likely include? a. Genetic diagnosis is not available as early as the second trimester. b. Genetic testing can identify presymptomatic conditions in children. c. Gene therapy can be used to repair missing genes with normal ones. d. Genetic agents may be used in the future to replace drugs.

a. Genetic diagnosis is not available as early as the second trimester. Genetic diagnosis is now possible very early in pregnancy. Genetic testing can now identify presymptomatic conditions in children and adults. Gene therapy can be used to replace or repair defective or missing genes with normal ones. Gene therapy has been used for a variety of disorders, including cystic fibrosis, melanoma, diabetes, HIV, and hepatitis. the potential exists for creation of increased intelligence and size through genetic intervention. Recent research using gene therapy shows promise for the generation of insulin-producing cells to cure diabetes. In the future, genetic agents may replace drugs, general surgery may be replaced by gene surgery, and genetic intervention may replace radiation.

A nurse is assessing a pregnant woman on a routine checkup. When assessing the womans gastrointestinal tract, which of the following would the nurse expect to find? (Select all that apply.) a. Hyperemic gums b. Increased peristalsis c. Complaints of bloating d. Heartburn e. Nausea

a. Hyperemic gums c. Complaints of bloating d. Heartburn e. Nausea Gastrointestinal system changes include hyperemic gums due to estrogen and increased proliferation of blood vessels and circulation to the mouth; slowed peristalsis; acid indigestion and heartburn; bloating and nausea and vomiting.

A woman in the 34th week of pregnancy says to the nurse, I still feel like having intercourse with my husband. The woman's pregnancy has been uneventful. The nurse responds based on the understanding that: a. It is safe to have intercourse at this time. b. Intercourse at this time is likely to cause rupture of membranes. c. There are other ways that the couple can satisfy their needs. d. Intercourse at this time is likely to result in premature labor.

a. It is safe to have intercourse at this time. Sexual activity is permissible during pregnancy unless there is a history of vaginal bleeding, placenta previa, risk of preterm labor, multiple gestation, incompetent cervix, premature rupture of membranes, or presence of infection.

Assessment of a pregnant woman reveals a pigmented line down the middle of her abdomen. The nurse documents this as which of the following? a. Linea nigra b. Striae gravidarum c. Melasma d. Vascular spiders

a. Linea nigra Linea nigra refers to the darkened line of pigmentationdown the middle of the abdomen in pregant women. Striae gravidarum refer to stretch marks, irregular reddish streaks on the abdomen, breasts, and buttocks. Melasma refers to the increased pigmentation on the face, also known as the "mask of pregnancy." Vascular spiders are small, spiderlike blood vessels that appear usually above the waist and on the neck, thorax, face, and arms.

A woman comes to the prenatal clinic suspecting that she is pregnant, and assessment reveals probable signs of pregnancy. Which of the following would be included as part of this assessment? (Select all that apply.) a. Positive pregnancy test. b. Ultrasound visualization of the fetus c. Auscultation of a fetal heart beat d. Ballottement e. Absence of menstruation f. Softening of the cervix

a. Positive pregnancy test. d. Ballottement f. Softening of the cervix Probable signs of pregnancy include a positive pregnancy test, ballottement, and softening of the cervix (Goodell's sign). Ultrasound visualization of the fetus, auscultation of a fetal heart beat, and palpation of fetal movements are considered positive signs of pregnancy. Absence of menstruation is a presumptive sign of pregnancy.

In a clients, seventh month of pregnancy, she reports feeling "dizzy", like "I'm going to pass out, when I lie down flat on my back." The nurse integrates which of the following into the explanation? a. Pressure of the gravid uterus on the vena cava b. A 50% increase in blood volume c. Physiologic anemia due to hemoglobin decrease d. Pressure of the presenting fetal part o n the diaphragm

a. Pressure of the gravid uterus on the vena cava The client is describing symptoms of supine hypotension syndrome, which occurs when the heavy gravid uterus falls back against the superior vena cava in the supine positon . The vena cava is compressed, reducing venous return, cardiac output, and blood pressure, with increased orthostasis. The increased blood volume and physiologic anemia are unrelated to the client's symptoms. Pressure on the diaphragm would lead to dyspnea.

A pregnant woman is scheduled to undergo percutaneous umbilical blood sampling. When discussing this test with the woman, the nurse reviews what can be evaluated with the specimens collected. Which of the following would the nurse include? (Select all that apply.) a. Rh incompatibility b. Fetal acidbase status c. Sex-linked disorders d. Enzyme deficiencies e. Coagulation studies

a. Rh incompatibility b. Fetal acidbase status e. Coagulation studies Specimens obtained via percutaneous umbilical cord sampling can be evaluated for coagulation studies, blood group typing, complete blood count, karyotyping, and blood gas analysis. Fetal infection, Rh incompatibility, and fetal acidbase status can be determined. Sex-linked disorders and enzyme deficiencies can be evaluated with chorionic villus sampling.

After teaching a group of students about the discomforts of pregnancy, the students demonstrate understanding of the information when they identify which as common during the first trimester? (Select all that apply.) a. Urinary frequency b. Breast tenderness c. Cravings d. Backache e. Leg cramps

a. Urinary frequency b. Breast tenderness c. Cravings Discomforts common in the first trimester include urinary frequency, breast tenderness, and cravings. Backache and leg cramps are common during the second trimester. Legs cramps are also common during the third trimester. 1st: 0 -13 weeks 2nd: 14 - 26 weeks 3rd: 27 - 40 weeks

A pregnant woman in her second trimester tells the nurse, "I've been passing a lot of gas and feel bloated." Which of the following suggestions would be helpful for the woman? Select all apply a. Watch how much beans and onions you eat. b. Limit the amount of fluid you drink with meals. c. Try exercising a little more. d. Some say that eating mints can help. e. Cut down on your intake of cheeses.

a. Watch how much beans and onions you eat. c. Try exercising a little more. d. Some say that eating mints can help. For gas and bloating, the nurse would instruct the woman to avoid gas-forming foods, such as beans, cabbage, and onions, as well as foods that have a high content of white sugar. Adding more biter to the diet, increasing fluid intake, and increasing physical exercise are also helpful in reducing flatus. In addition, reducing the amount of swallowed air when chewing gum or smoking will reduce gas build-up. Reducing the intake of carbonated beverages and cheese and eating mints can also help reduce flatulence during pregnancy.

After teaching a group of students about fetal development, the instructor determines that the teaching was successful when the students identify which of the following as providing the barrier to other sperm after fertilization? a. Zona pellucida b. Zygote c. Cleavage d. Morula

a. Zona pellucida The zona pellucida is the clear protein layer that acts as a barrier to other sperm once one sperm enters the ovum for fertilization. The zygote refers to the union of the nuclei of the ovum and sperm resulting in the diploid number of chromosomes. Cleavage is another term for mitosis. The morula is the result of four cleavages leading to 16 cells that appear as a solid ball of cells. The morula reaches the uterine cavity about 72 hrs after fertilization.

A couple comes to the clinic for preconception counseling and care. As part of the visit, the nurse teaches the couple about fertilization and initial development, stating that the zygote formed by the union of the ovum and sperm consists of how many chromosomes? a. 22 b. 23 c. 44 d. 46

d. 46 with fertilization, the ovum, containing 23 chromosomes, and the sperm, containing 3 chromosomes, join, forming a zygote with a diploid number or 46 chromosomes.

After the nurse describes fetal circulation to a pregnant woman, the woman asks why her fetus has a different circulation pattern than hers. In planning a response, the nurse integrates understanding of which of the following? a. Fetal blood is thicker than that of adults and needs different pathways. b. Fetal circulation carries highly oxygenated blood to vital areas first. c. Fetal blood has a higher oxygen saturation and circulates more slowly. d. Fetal heart rates are rapid and circulation time is double that of adults.

b. Fetal circulation carries highly oxygenated blood to vital areas first. Fetal circulation functions to carry highly oxygenated blood to vital areas first while shunting it away from less vital ones. Fetal blood is not thicker than that of adults. Large volumes of oxygenated blood are not needed because the placenta essentially takes over the functions of the lung and liver during fetal life. Although fetal R normally range from 120 to 160 beats per minute, circulation time is not doubled.

When describing the structures involved in fetal circulation, the nursing instructor describes which structure as the opening between the right and left atrium? a. Ductus venosus b. Foramen ovale c. Ductus arteriosus d. Umbilical artery

b. Foramen ovale the foramen ovale is the opening between the right and left atrium. The ductus venous connects the umbilical vein to the inferior vena carva. The ductus arterious connects the main pulmonary artery to the aorta. The umbilical artery carries blood to the placenta.

During a routine prenatal visit, a client, 36 weeks pregnant, states she has difficulty breathing and feels like her pulse rate is really fast. The nurse finds her pulse to be 100 beats per minute (increased from baseline readings of 70 to 74 beats per minute. and irregular, with bilateral crackles in the lower lung bases. Which nursing diagnosis would be the priority for this client? a. Ineffective tissue perfusion related to supine hypotensive syndrome b. Impaired gas exchange related to pulmonary congestion c. Activity intolerance related to increased metabolic requirements d. Anxiety related to fear of pregnancy outcome

b. Impaired gas exchange related to pulmonary congestion Typically, heart rate increases by approximately 10 to 15 beats per minute during pregnancy and the lungs should be clear. Dyspnea may occur during the third trimester as the enlarging uterus presses on the diaphragm. However, the findings described indicate that the woman is experiencing impaired gas exchange. There is no evidence to support problems with tissue perfusion, activity, or anxiety.

A nursing instructor is preparing a teaching plan for a group of nursing students about the potential for misuse of genetic discoveries and advances. Which the following would the instructor most likely include? a. Gene replacement therapy for defective genes b. Individual risk profiling and confidentiality c. Greater emphasis on the causes of diseases d. Slower diagnosis of specific disease

b. Individual risk profiling and confidentiality

A woman comes to the clinic complaining of a vaginal discharge. The nurse suspects that the client has an infection. When gathering additional information, which of the following would the nurse be least likely to identify as placing the client at risk for an infection? a. Recent antibiotic therapy for an upper respiratory infection b. Last menstrual period about 5 days ago c. Weekly douching d. Frequent use of feminine hygiene sprays.

b. Last menstrual period about 5 days ago The vagina has an acidic environment, which protects it against ascending infections. Antibiotic therapy, douching, perineal hygiene sprays, and deodorants upset the acid balance within the vaginal environment and can predispose women to infections. Menstrual period about 5 days ago is not considered a risk factor.

A nurse is assessing a child with Klinefelter's syndrome. Which of the following would the nurse expect to assess? (Select all that apply.) a. Gross mental retardation b. Long arms c. Profuse body hair d. Gynecomastia e. Enlarged testicles

b. Long arms d. Gynecomastia Manifestations of Klinefelter's syndrome include mild mental retardation, small testicles, infertility, long arms and legs, gynecomastia, scant facial and body hair, and decreased libido.

When describing the hormones involved in the menstrual cycle, a nurse identifies which hormone as responsible for initiating the cycle? a. Estrogen b. Luteinizing hormone c. progesterone d. Prolactin

b. Luteinizing hormone

When describing the male sexual response to a group of students, the instructor determines that the teaching was successful when they identify emission as which of the following? a. Semen forced through the urethra to the outside b. Movement of sperm from the testes and fluid into the urethras c. Dilation of the penile arteries with increased blood flow to the tissues. d. Bodys return to the physiologic nonstimulated state

b. Movement of sperm from the testes and fluid into the urethras Emission refers to the movement of sperm from the testes and fluids from the accessory glands into the urethra, where it is mixed to form semen. As the urethra fills with semen, the base of the erect penis contracts, thus increases pressure. This pressure force the semen through the urethra to the outside (ejaculation). Dilation of the penile arteries with increased blood flow describes erection. The body's return to the physiologic nonstimulated state describes resolution.

A nurse is teaching a class on X-linked recessive disorders. Which of the following statements would the nurse most likely include? a. Males are typically carriers of the disorders. b. No male-to-male transmission occurs. c. Daughters are more commonly affected with the disorder. d. Both sons and daughters have a 50% risk of the disorder.

b. No male-to-male transmission occurs. Most X-linked disorders demonstrate a recessive pattern of inheritance. Males are more affected than females. A male has only one X chromosome and all the genes on his X chromosome will be expressed, whereas a female will usually need oth X chromosomes to carrry the disease. There is no male-to-male transmission (since not X chromosome from the male is transmitted to male offspring), but any man who is affected will have carrier daughter. if a woman is carrier, ther is a 50% chance that her sons will be affected and a 50% chance that her daughters will be carrieers.

A clients maternal serum alpha-fetoprotein (MSAFP. level was unusually elevated at 17 weeks. The nurse suspects which of the following? a. Fetal hypoxia b. Open spinal defects c. Down syndrome d. Maternal hypertension

b. Open spinal defects Elevated MSAFP levels are associated with open neural tube defects. Fetal hypoxia would be noted with fetal heart rate tracings and via nonstress and contraction stress testing. MSAFP in conjunction with marker screening tests would be more reliable for detecting Down syndrome. Maternal hypertension would be noted via serial blood pressure monitoring.

After teaching a group of students about the different perinatal education methods, the instructor determines that the teaching was successful when the students identify which of the following as the Bradley method? a. Psychoprophylactic method b. Partner-coached method c. Natural childbirth method d. Mind prevention method

b. Partner-coached method The Bradley meth is also a partner-coached method that uses various exercises and slow, controlled abdominal breathing to accomplish relaxation and active participation of the partner as labor coach. The Dick-Read method is referred to as natural childbirth. Dick-Read believed that prenatal instruction was essential for pain relief and that emotional factors during labor interfered with the normal labor progression. The woman achieves relaxation and reduces pian by arming herself with the knowledge of normal childbirth and using abdominal breathing during contractions.

Assessment of a pregnant woman reveals oligohydramnios. The nurse would be alert for the development of which of the following? a. Maternal diabetes b. Placental insufficiency c. Neural tube defects d. Fetal gastrointestinal malformations

b. Placental insufficiency A deficiency of amniotic fluid, oligohydramnios, is associated with uteroplacental insufficiency and fetal renal abnormalities. Excess amniotic fluid is associated with maternal diabetes, neural tube defects, and malformations of the gastrointestinal tract and central nervous system.

After teaching a class on the stages of fetal development, the instructor determines that the teaching was successful when the students identify which of the following as a stage? (Select all that apply.) a. Placental b. Preembryonic c. Umbilical d. Embryonic e. Fetal

b. Preembryonic c. Umbilical e. Fetal The three stages of fetal development are the preembryonic, embryonic, and fetal stage.

A nurse is reviewing the medical record of a pregnant woman and notes that she is gravid II. The nurse interprets this to indicate the number of: a. Deliveries b. Pregnancies c. Spontaneous abortions d. Pre-term births

b. Pregnancies Gravid refers to a pregnant woman - gravid I during the first pregnancy, gravid II during the second pregnancy, and so on. Para refers to the number deliveries at 20 weeks or greater that a woman has, regardless of whether the newborn is bron alive or dead. A would be the number of abortions P would be the number preterm births

When describing the menstrual cycle to a group of young women, the nurse explains that estrogen levels are highest during which phase of the endometrial cycle? a. Menstrual b. Proliferative c. Secretory d. Ischemic

b. Proliferative Estrogen levels are the highest during the proliferative phase of the endometrial cycle, when the endometrial glands enlarge in response to increasing amounts of estrogen. Progesterone is the predominant hormone of the secretory phase. Levels of estrogen and progesterone drop sharply during the ischemic phase and fall during the menstrual phase.

When describing perinatal education to a pregnant woman and her partner, the nurse emphasizes that the primary goal of these classes is to: a. Equip a couple with the knowledge to experience a pain-free childbirth b. Provide knowledge and skills to actively participate in birth and parenting c. Eliminate anxiety so that they can have an uncomplicated birth d. Empower the couple to totally control the birth process

b. Provide knowledge and skills to actively participate in birth and parenting The primary focus of perinatal education is to provide information and support to clients and their families to foster a more active role in the upcoming birth. It also includes preparation for breastfeeding, infant care, transition to new parenting roles, relationships skills, family health promotion, and sexuality. Some methods of childbirth education focus on pain-free childbirth. Information provided in childbirth education classes helps to minimize anxiety and provide the couple with control over the situation, but elimination of anxiety or total control is unrealistic

A primiparous client is being seen in the clinic for her first prenatal visit. It is determined that she is 11 weeks pregnant. The nurse develops a teaching plan to educate the client about what she will most likely experience during this period. Which of the following would the nurse include? a. Ankle edema b. Urinary frequency c. Backagche d. Hemorrhoids

b. Urinary frequency The client is in her first trimester and would most likely experience urinary frequency as the growing uterus presses on the bladder. Ankle edema, backache, and hemorrhoids would be more common during the later stages of pregnancy.

A pregnant woman comes to the clinic and tells the nurse that she has been having a whitish vaginal discharge. The nurse suspects vulvovaginal candidiasis based on which assessment finding? a. Fever b. Vaginal itching c. Urinary frequency d. Incontinence

b. Vaginal itching Vaginal secretions become more acidic, white, and thick during pregnancy. Most women experience an increase in a whitish vaginal discharge, called leukorrhea. This is normal except when it is accompanied by itching and irritation, possible suggesting Candida albicans, a monilial vaginitis, which is a very common occurrence in this glycogen-rich environment. Fever would suggest a more serious infection. Urinary frequency occurs commonly in the first and third trimester. Incontinence would not be associated with vulvovaginal candidiasis, would require additional evaluation.

A group of nursing students are reviewing information about the male reproductive structures. The students demonstrate understanding of the information when they identify which of the following as accessory organs? (Select all that apply.) a. Testes b. Vas deferens c. Bulbourethral glands d. prostate gland e. Penis

b. Vas deferens c. Bulbourethral glands d. prostate gland The organs of the male reproductive system include the two testes (where sperm cells and testosterone are made), the penis, the scrotum and the accessory organs (epididymis, vas deferens, seminal vesicles, ejaculatory duct, urethra, bulbourethral glands, and prostate gland).

The nurse is discussing the insulin needs of a primaparous client with diabetes who has been using insulin for the past few years. The nurse informs the client that her insulin needs will increase during pregnancy based on the nurses understanding that the placenta produces a. hCG, which increases maternal glucose levels b. hPl, wich deceases the effectiveness of insulin c. Estriol, which interferes with insulin crossing the placenta d. Relaxin, which decreases the amount of insulin produced

b. hPl, wich deceases the effectiveness of insulin hPl acts as an antagonist to insulin, so the mother must produce more insulin to overcome this resistance. if the mother has diabetes, then her insulin need would most likely increase to meet this demand. hCG does not affect insulin and glucose level. Estrogen, not estriol, is believed to oppose insulin. In addition, insulin does not cross the placenta. Relaxin is not associated with insulin resistance.

A nurse strongly encourages a pregnant client to avoid eating swordfish and tilefish because these fish contain which of the following? a. Excess folic acid, which could increase the risk for neural tube defects b. Mercury, which could harm the developing fetus if eaten in large amounts c. Lactose, which leads to abdominal discomfort, gas, and diarrhea d. Low-quality protein that does not meet the woman's requirements

b. mercury, which could harm the developing fetus if eaten in large amounts Nearly all fish and shellfish contain traces of mercury and some contain higher levels of mercury that may harm the developing fetus if ingested by pregnant women in large amounts. Among these fish are shark, swordfish, king mackerel, and tilefish. Folic acid is found in dark green vegetables, baked beans, black-eyed peas, citrus fruits, peanuts, and liver. Folic acid supplements are needed to prevent neural tube defects. Women who are lactose intolerant experience abdominal discomfort, gas, and diarrhea if they ingest foods containing lactose. Fish and shellfish are an important part of a healthy diet because they contain high-quality proteins, are low in saturated fat, and contain omega-3 fatty acids.

On the first prenatal visit, examination of the womans internal genitalia reveals a bluish coloration of the cervix and vaginal mucosa. The nurse records this finding as: a. Hegar's sign b. Goodell's sign c. Chadwick's sign d. Homan's sign

c. Chadwick's sign Chadwick's sign refers to the bluish coloration of the cervix and vaginal mucosa. Hegar's sign refers to softening of the isthmus. a. Hegar's sign: softening of the uterus b. Goodell's sign: oftening of the cervix d. Homan's sign: indicates pain on dorsiflexion of the food.

When preparing a woman for an amniocentesis, the nurse would instruct her to do which of the following? a. Shower with an antiseptic scrub. b. Swallow the perprocedure sedative. c. Empty her bladder. d. Lie on her left side.

c. Empty her bladder. Before an amniocentesis, the woman should empty her bladder to reduce the risk of bladder puncture during the procedure. Showering with an antiseptic scrub and pre-procedural sedation are not necessary. The woman usually is positioned in a way that proves an adequate pocket of amniotic fluid on ultrasound.

After teaching a group of students about female reproductive anatomy, the instructor determines that the teaching was successful when the students identify which of the following as the site of fertilization? a. Vagina b. Uterus c. Fallopian tubes d. Vestibule

c. Fallopian tubes Fertilization occurs in the distal portion of the fallopian tubes. The lining of the uterus is shed with menstruation. The vagina connects the external genitalia to the uterus. The vestibule is an oval area enclosed by the labia minora laterally.

A nurse is working with a pregnancy woman to schedule follow-up visits for her pregnancy. Which statement by the woman indicates that she understands the scheduling? a. I need to make visits every 2 months until I'm 36 weeks pregnant. b. Once I get to 28 weeks, I have to come twice a month. c. From now until I'm 28 weeks, I'll be coming once a month. d. I'll make sure to get a day off every 2 weeks to make my visits.

c. From now until I'm 28 weeks, I'll be coming once a month. Continuous prenatal care is important for a successful pregnancy outcome. The recommended follow-up visit schedule for a healthy pregnant woman is as follows; every 4 weeks up to 28 weeks (7 months); every 2 weeks from 29 to 36 weeks; every week from 37 weeks to birth

Prenatal testing is used to assess for genetic risks and to identify genetic disorders. In explaining to a couple about an elevated alpha-fetoprotein screening test result, the nurse would discuss the need for: a. Special care needed for a Down syndrome infant b. A more specific determination of the acidbase status c. Further, more definitive evaluations to conclude anything d. Immediate termination of the pregnancy based on result

c. Further, more definitive evaluations to conclude anything

A pregnant woman is flying across the country to visit her family. After teaching the woman about traveling during pregnancy, which statement indicates that the teaching was successful? a. I'll sit in a window seat so I can focus on the sky to help relax me. b. I won't drink too much fluid so I don't have to urinate so often. c. I'll get up and walk around the airplane about every 2 hrs. d. I'll do some upper arm stretches while sitting in my seat.

c. I'll get up and walk around the airplane about every 2 hrs. When traveling by airplane, the woman should get up and walk about the plane every 2 hours to promote circulation. An aisle seat is recommended so that she can have easy access to the aisle. drinking water throughout the flight is encouraged to maintain hydration. Calf-tensing exercises are important to improve circulation to the lower extremities.

A pregnant woman has a rubella titer drawn on her first prenatal visit. The nurse explains that this test measures which of the following? a. Platelet level b. Rh status c. Immunity to German measles d. Red blood cell count

c. Immunity to German measles A rubella titer detects antibodies for the virus that causes German measles. If the titer is 1:8 or less, the woman is not immune and requires immunization after birth. Platelet level and red blood cell count would be determined by a complete blood count. Rh status would be determined by blood typing.

Which of the following changes in the musculoskeletal system would the nurse mention when teaching a group of pregnant women about the physiologic changes of pregnancy? a. Ligament tightening b. Decreased swayback c. Increased lordosis d. Joint contraction

c. Increased lordosis With pregnancy, the woman's center of gravity shifts forward, requiring a realignment of the spinal curvatures. There is an increase in the normal curve (lordosis). Ligaments o f the sacroiliac joints and pubis symphysis soften and stretch. Increased swayback and an upper spine extension to compensate for the enlarging abdomen occur. Joint relaxation and increased mobility occur due to the influence of the hormones relaxin and progesterone.

A woman in her second trimester comes for a follow-up visit and says to the nurse, "I feel like I'm on an emotional roller-coaster." Which response by the nurse would be most appropriate? a. How often has this been happening to you? b. Maybe you need some medication to level things out. c. Mood swings are completely normal during pregnancy. d. Have you been experiencing any thoughts of harming yourself.?

c. Mood swings are completely normal during pregnancy.

A woman suspecting she is pregnant asks the nurse about which signs would confirm her pregnancy. The nurse would explain that which of the following would confirm the pregnancy? a. Absence of menstrual period b. Abdominal enlargmenet c. Palpable fetal movement d. Morning sickness

c. Palpable fetal movement Only positive signs of pregnancy would confirm a pregnancy. The positive signs of pregnancy confirm that a fetus is growing in the uterus. Visualizing the fetus by ultrasound, palpating for fetal movements, and hearing a fetal hearbeat are all signs that make the pregnancy a certainty. Absence of menstrual period and morning sickness are presumptive signs, which can be due to conditions other than pregnancy. Abdominal enlargement is a probable sign.

The nurse is assessing a pregnant woman in the second trimester. Which of the following tasks would indicate to the nurse that the client is incorporating the maternal role into her personality? a. The woman deomonstrates concern for herself and her fetus as a unit. b. The client identifies what she must give up to assume her new roles. c. The woman acknowledges the fetus as a separate entity within her. d. The client demonstrates unconditional acceptance without rejection.

c. The woman acknowledges the fetus as a separate entity within her. Incorporation of the maternal role into her personality indicates acceptance by the pregnant woman. In doing so, the woman becomes able to identify the fetus as a separate individual. Demonstrating concern for herself and her fetus as a unit is associated with introversion and more commonly occurs during the third trimester. Identification of what the mother must give up to assume the new role occurs during the first trimester. Demonstrating unconditional acceptance without rejection occurs during the third trimester.

A pregnant woman in the 36th week of gestation complains that her feet are quite swollen at the end of the day. After careful assessment, the nurse determines that this is an expected finding at this stage of pregnancy. Which intervention would be most appropriate for the nurse to suggest? a. Limit your intake of fluids. b. Eliminate salt from your diet. c. Try elevating your legs when you sit. d. Wear Spandex-type full-length pants.

c. Try elevating your legs when you sit. The PT is experiencing dependent edema due to the effect of gravity and increased capillary permeability caused by elevated hormone levels and increased blood volume, and accompanied by sodium and water retention. The best suggestion would be to encourage the PT to elevate her legs when sitting to promote venous return and minimize the effects of gravity. Neither fluids nor salt should be limited or eliminated.

Which female reproductive tract structure would the nurse describe to a group of young women as containing rugae that enable it to dilate during labor and birth? a. Cervix b. Fallopian tube c. Vagina d. Vulva

c. Vagina The vagina is a tubular, fibromuscular organ lined with mucous membrane that lies in a series of transverse folds called rugae. These rugae allow for extreme dilation of the canal during labor and birth. The cervix, the lower portion of the uterus, is composed of fibrous connective tissue that dilates during labor. The fallopian tube transports the ovum from the ovary to the uterus. The vulva is a collective term used to refer to the external female reproductive organs (mons pubis, labia majora and minora, clitoris, vestibular structures, and perineum).

A nursing instructor is teaching a class to a group of students about pregnancy, insulin, and glucose. Which of the following would the instructor least likely include as opposing insulin? a. Prolactin b. Estrogen c. Progesterone d. Cortisol

d. Cortisol Prolactin, estrogen, and progesterone are all thought to oppose insulin. As a result, glucose is lees likely to enter the mother's cells and is more likely to cross over the placenta to the fetus. Afte first trimester, hPL from the placenta and steroids (cortisol) from the adrenal cortex act against insulin. hPL acts as an antagonist against maternal insulin, and thus more insulin must be secreted to counteract the increasing levels of hPL and cortisol during the last half of pregnancy.

When assessing a woman at follow-up prenatal visits, the nurse would anticipate which of the following to be performed? a. Hemoglobin and hematocrit b. Urine for culture c. Fetal ultrasound d. Fundal height measurement

d. Fundal height measurement

When teaching a pregnant client about the physiologic changes of pregnancy, the nurse reviews the effect of pregnancy on glucose metabolism. Which of the following would the nurse include as the underlying reason for the effect? a. Pancreatic function is affected by pregnancy. b. Glucose is utilized more rapidly during a pregnancy. c. The pregnant woman increases her dietary intake. d. Glucose moves through the placenta to assist the fetus.

d. Glucose moves through the placenta to assist the fetus. The growing fetus has large needs for glucose, amino acids, and lipids, placing demands on maternal glucose stores. During the first half of pregnancy, much of the maternal glucose is diverted to the growing fetus. The pancreas continues to function during pregnancy. However, the placental hormones can affect maternal insulin levels. The demand for glucose by the fetus during pregnancy is high, but it is not necessarily used more rapidly. Placental hormones, not the woman's dietary intake, play a major role in glucose metabolism during pregnancy.

A pregnant client in her second trimester has a hemoglobin level of 11 g/dL. The nurse interprets this as indicating which of the following? a. Iron-deficiency anemia b. A multiple gestation pregnancy c. Greater-than -expected weight gain d. Hemodilution of pregnancy

d. Hemodilution of pregnancy During pregnancy, the red blood cell count increases along with an increase in plasma volume. However, there is a greater increase in the plasma volume as a result of hormonal factors and sodium and water retentions. Thus, the plasma increase exceeds the increase in RBCs, resulting in hemodilution of pregnancy, which is also called physiologic anemia of pregnancy. Changes in maternal iron levels would be more indicative of an iron-deficiency anemia. Although anemia may be present with a multiple gestation, an ultrasound would be a more reliable method of identifying it. Weight gain does not correlate with hemoglobin levels.

A pregnant woman needs an update in her immunizations. Which of the following vaccinations would the nurse ensure that the woman receives? a. Measles b. Mumps c. Rubella d. Hepatitis B

d. Hepatitis B Hepatitis B vaccine should be considered during pregnancy. Immunizations for measles, mumps, and rubella are contraindicated during pregnancy.

When assessing a pregnant woman in her last trimester, which question would be most appropriate to use to gather information about weight gain and fluid retention? a. What is your usual dietary intake for a typical day? b. What size maternity clothes are you wearing now? c. How puffy does your face look by the end of a day? d. How swollen do your ankles appear before you go to bed?

d. How swollen do your ankles appear before you go to bed? Edema, especially in the dependent areas such as the legs and feet, occurs throughout the day due to gravity. It improves after a night's sleep. Therefore, questioning the PT about ankle swelling would provide the most valuable information. a. Asking about her usual dietary intake would be valuable in assessing complaints of heartburn and indigestion. b. The size of maternity clothing may provide information about weight gain but would have little significance for fluid retention. c. Swelling in the face may suggest preeclampsia, especially if it is accompanied by dizziness, blurred vision, headaches, upper quadrant pain, or nausea.

After teaching a pregnant woman about the hormones produced by the placenta, the nurse determines that the teaching was successful when the woman identifies which hormone produced as being the basis for pregnancy tests? a. Human placental lactogen (hPL) b. Estrogen (estriol) c. Progesterone (progestin) d. Human chorionic gonadotropin (hCG)

d. Human chorionic gonadotropin (hCG) The placenta produces hCG, which is the basis for pregnancy tests. This hormone preserves the corpus luteum and its progesterone production so that the endometrial lining is maintained. Human placental lactogen modulates fetal and maternal metabolism and participates in the development of the breasts for lactation. Estrogen causes enlargement of the woman's breasts, uterus, and external genitalia and stimulates myometrial contractility. Progesterone maintains the endometrium.

During a nonstress test, when monitoring the fetal heart rate, the nurse notes that when the expectant mother reports fetal movement, the heart rate increases 15 beats or more above the baseline. The nurse interprets this as: a. Variable decelerations b. Fetal tachycardia c. A nonreactive pattern d. Reactive pattern

d. Reactive pattern A reactive non stress test indicates fetal activity, as evidenced by acceleration of the fetal heart rate by at least 15 ppm for at least 15 seconds within a 20 minute recording period. If this does not occur, the test is considered nonreactive. An increase in the fetal heart rate does not indicate variable decelerations. Fetal tachycardia would be noted as a heart rate greater than 160 bpm

A woman comes to the clinic complaining that she has little sexual desire. As part of the clients evaluation, the nurse would anticipate the need to evaluate which hormone level? a. Progesterone b. Estrogen c. Gonadotropin-releasing hormone d. Testosterone

d. Testosterone Testosterone is needed to stimulate sexual desire in women. Thus, an evaluation of this level would be done. Progesterone is often called the hormone of pregnancy because of its calming effect (reduction in uterine contractions) on the uterus, allowing pregnancy to be maintained. Estrogen is the predominant hormone at the end of the follicular phase. Gonadotropin-releasing hormone induces the release of FSH and LH to assist with ovulation.

The nurse teaches a primigravida client that lightening occurs about 2 weeks before the onset of labor. The mother will most likely experience which of the following at that time? a. Dysuria b. Dyspnea c. Constipation d. Urinary frequency

d. Urinary frequency Lightening refers to the descent to the fetal into the pelvis and engagement. With this descent, pressure on the diaphragm decreases, easing breathing, but pressure on the bladder increases, leading to urinary frequency. Dysuria might indicate a urinary tract infection. Constipation may occur throughout pregnancy due to decreased peristalsis, but it is unrelated to lightening.


संबंधित स्टडी सेट्स

MUS Geology- Chapter 1 Test (No T/F)

View Set

head and spinal cord injury quiz questions

View Set

CPR: Heart Attack and Cardiac Arrest

View Set

Chapter 6: Etruscan and Roman Architecture

View Set

Physiology - Renal System Lecture 2

View Set

Peds Exam 1 (Jordan) with PANCE questions

View Set

High School Health (UNIT 3: SOCIAL AND MENTAL HEALTH) SOCIAL HEALTH

View Set

Chapt. 51: Assessment & managament of pts with diabetes

View Set

Law and Ethics CH. 4 Evolve practice questions

View Set

IAS 107 Midterm 1 Multiple Choice

View Set